SSC GD 15 Feb 2019 Shift-III Previous Year Paper

SSC GD 15th Feb 2019 Shift-III 

Reasoning 

Instructions 

For the following questions answer them individually 

Q. 1 Select the Venn diagram that best represents the given set of classes. 

Stationery, Pen, Eraser 

Answer: (C) 

Explanation: 

Pen and Eraser are both parts of stationary but are completely different from each other, hence the venn diagram that best describes above relationship is : 

=> Ans – (C) 

 

Q. 2 Select the option that will correctly replace the question mark (?) in the given pattern. 

(A)

(B) 216 

(C) 217 

(D) 36 

Answer: (C) 

Explanation: 

(n3 + 1) n 

Starting from 2 and going in anti clockwise direction, the pattern followed is , where is a natural number. (1)3 + 1 = 2 

(2)3 + 1 = 9 

(3)3 + 1 = 28 

(4)3 + 1 = 65 

(5)3 + 1 = 126 

(6)3 + 1 = 217 

=> Ans – (C) 

 

Q. 3 Select the option that will correctly replace the question mark (?) in the series. 

7, 19, 36, 63, ? 

(A) 125 

(B) 105 

(C) 115 

(D) 95 

Answer: (B) 

Explanation: 

The pattern followed is that numbers with successive multiples of 5 between them are added. 7 + 12 = 19 

19 + 17 = 36 

36 + 27 = 63 

63 + 42 = 105 

=> Ans – (B) 

 

Q. 4 Select the option that will correctly replace the question mark (?) in the series. 

107, 103, 87, 51, ? 

(A) -13 

(B) 15 

(C) 13 

(D) -15 

Answer: (A) 

 

Q. 5 In a certain code, COMPUTER is written as CEMOPRTU. How will CUPBOARD be written in that code? 

(A) ABCDOPRU 

(B) ABCDORPU 

(C) ABCDOQRU 

(D) ABDCOPRU 

Answer: (A) 

Explanation: 

COMPUTER is written as CEMOPRTU 

The pattern followed is that the letters are arranged in ascending order as they appear in the English alphabetical order. Similarly, CUPBOARD : ABCDOPRU 

=> Ans – (A) 

 

Q. 6 Choose the option that would follow next in the given figure series. 

Answer: (C) 

Explanation: 

The pattern followed is that the little gap between the two triangles in the first figure is pointing at top left and is moving in anti clockwise direction, i.e. in the second figure is pointing at left side, then bottom left and then at bottom, thus in the missing figure, it will face bottom right as shown in third figure. 

=> Ans – (C) 

 

Q. 7 The statements below are followed by two conclusions labeled I and II. Assuming that the information in the statements is true, even if it appears at variance with generally established facts, decide which conclusion(s) logically and definitely follow(s) from the information given in the statements. 

Statements: 

1) Some tables are sofas. 

2) Some sofas are papers. 

Conclusion 

I. Some sofas are tables. 

II. Some papers are sofas. 

(A) Only conclusion I follows. 

(B) Only conclusion II follows. 

(C) Both conclusions follow. 

(D) Neither conclusion I nor conclusion II follows. 

Answer: (C) 

Explanation: 

The venn diagram for above statements is : 

Conclusion 

I. Some sofas are tables = true 

II. Some papers are sofas = true 

Thus, both conclusions follow. 

=> Ans – (C) 

 

Q. 8 Select the option that will correctly replace the question mark (?) in the series. 

RGN, PIM, NKL, ? 

(A) LMK 

(B) LMJ 

(C) LVW 

(D) LWU 

Answer: (A) 

Explanation: 

Expression : RGN, PIM, NKL, ? 

The pattern followed in each term of the series is : 

1st term : R (-2) = P (-2) = N (-2) = L 

2nd term : G (+2) = I (+2) = K (+2) = M 

3rd term : N (-1) = M (-1) = L (-1) = K 

Thus, missing term is : LMK 

=> Ans – (A) 

 

Q. 9 Select the option that is related to the third number in the same way as the second number is related to the first number. 

54 : 6 :: 42 : ? 

(A)

(B)

(C)

(D)

Answer: (D) 

 

Q. 10 In a certain code, JOY is written as 150. How will FEED be written in that code? 

(A) 80 

(B) 90 

(C) 50 

(D) 20 

Answer: (A) 

 

Q. 11 Eight friends A, B, C, D, E, F, G and H are sitting around a circular table facing each other for a lunch. A is opposite D and third to the right of B. G is between A and F.H is to the right of A. E is between C and D. Who is sitting opposite G? 

(A)

(B)

(C)

(D)

Answer: (D) 

Explanation: 

A is opposite D and third to the right of B. Also, E is between C and D. 

=> B is to the immediate right of D. 

G is between A and F, => G is to the immediate left of A. 

H is to the immediate right of A. 

Thus, E is sitting opposite G. 

=> Ans – (D) 

 

Q. 12 The statements below are followed by two conclusions labeled I and II. Assuming that the information in the statements is true, even if it appears at variance with generally established facts, decide which conclusion(s) logically and definitely follow(s) from the information given in the statements. 

Statements: 

1) All spiders are snails. 

2) All molars are snails. 

Conclusion 

I. No spider is a molar. 

II. Some spiders are molars. 

(A) Only conclusion II follows. 

(B) Only conclusion I follows. 

(C) Either conclusion I or conclusion II follows. 

(D) Both conclusions follow. 

Answer: (C) 

Explanation: 

The venn diagram for above statements is : 

Conclusion 

I. No spider is a molar = may or may not be true 

II. Some spiders are molars = may or may not be true 

Thus, either conclusion I or II follows. 

=> Ans – (C) 

 

Q. 13 Choose the option that most closely resembles the mirror image of the given figure when the mirror is placed at the right side.

Answer: (A) 

Explanation: 

Image in third option is the exact replica of question figure, hence it is eliminated. Now, the vertical shape of circular will remain intact, hence second and fourth options are also not possible. Thus, first option is the correct mirror image. 

=> Ans – (A) 

 

Q. 14 Choose the odd number out of the given options. 

(A) 17 

(B) 89 

(C) 63 

(D) 37 

Answer: (C) 

Explanation: 

17, 89 and 37 are prime numbers, hence 63 is the odd one. 

=> Ans – (C) 

 

Q. 15 Choose the odd one out of the given options. 

(A) Legs 

(B) Stomach 

(C) Heart 

(D) Kidneys 

Answer: (A) 

Explanation: 

Apart from legs, all other body parts are internal organs (which cannot be seen), hence legs are the odd one. => Ans – (A) 

 

Q. 16 Seven friends O, P, Q. R, S, T and U are watching a movie sitting in a row. P is sitting at one extreme end. Q is sitting to the immediate left of S. P is sitting second to the right of T. U is not sitting at any extreme end. O is sitting between R and T. Who is sitting in the middle? 

(A)

(B)

(C)

(D)

Answer: (C) 

 

Q. 17 A square transparent sheet with a pattern is given. How will the pattern appear when the transparent sheet is folded along the dotted line? 

Answer: (D) 

 

Q. 18 Choose the odd one out of the given options. 

(A) CJK 

(B) EHK 

(C) FHJ 

(D) KOJ 

Answer: (D) 

Explanation: 

In the first three options, the letters are in increasing order, i.e. as they appear in the English alphabetical order, but in last option, O appears after both J and K, hence it is the odd one. 

=> Ans – (D) 

 

Q. 19 Select the option that is related to the third term in the same wayas the second term is related to the first term. 

Ichthyology : Fishes :: Palaeography : ? 

(A) Writings 

(B) Soil 

(C) Shells 

(D) Moon 

Answer: (A) 

Explanation: 

Ichthyology is the branch of zoology that deals with fishes. Similarly, Palaeography is the study of ancient writing systems. => Ans – A 

 

Q. 20 Find out the two signs to be interchanged to make the following equation correct. 

27 + 27 × 27 − 27 ÷ 27 = −701 

(A) ÷ and × 

(B) + and ÷ 

(C) − and ÷ 

(D) × and − 

Answer: (A) 

 

Q. 21 Choose the option in which the figure marked ‘x’ is embedded. (Rotation is not allowed) 

Answer: (A) 

 

Q. 22 Select a figure from amongst the four alternatives that when placed in the blank space (?) of figure X will complete the pattern. (Rotation is not allowed). 

Answer: (D) 

 

Q. 23 J, K L, M, N and O are six teachers. Each one teachesa different subject out of Hindi, English, Math, Science, Social Science and Arts, not necessarily in the same order. Each of them teaches on only one day, from Mondayto Saturday, not necessarily in the same order. J teaches Science on Wednesday. O teaches Math second after J. K teaches on the first day of the week, but teaches neither Hindi nor English. M teaches English before N and L teaches Arts before J. Which subject is taught on Thursday? 

(A) Science 

(B) Arts 

(C) English 

(D) Social Science 

Answer: (C) 

 

Q. 24 Select the option that is related to the third term in the same way as the second term is related to the first term. 

BFH : EIK :: GKM : ? 

(A) JNP 

(B) KMG 

(C) MKG 

(D) IKM 

Answer: (A) 

 

Q. 25 Select the option that is related to the third term in the same way as the second term is related to the first term. 

Innocent : Guilty :: Moisten : ? 

(A) Drench 

(B) Dried 

(C) Desire 

(D) Wet 

Answer: (B) 

Explanation: 

Expression = Innocent : Guilty :: Moisten : ? 

The first two words are opposite to each other, similarly antonym of moisten is dried. => Ans – (B) 

General knowledge 

Instructions 

For the following questions answer them individually 

Q. 26 Who started the newspaper Mook Nayak in the year 1920? 

(A) Dadabhai Naoroji 

(B) Jyotiba Phule 

(C) B. R Ambedkar 

(D) Mahadeo Govind Ranade 

Answer: (C) 

 

Q. 27 What was the official mascot of the third Asian Para Games 2018? 

(A) Momo 

(B) Appu 

(C) Ezomon 

(D) Chim Yen 

Answer: (A) 

 

Q. 28 Which part protects an unopened flower? 

(A) Sepal 

(B) Stamen 

(C) Petal 

(D) Anther 

Answer: (A) 

 

Q. 29 Which schedule was added to the Indian Constitution in 1992 as a result of 73rd amendment? 

(A) 12th 

(B) 9th 

(C) 8th 

(D) 11th 

Answer: (D) 

 

Q. 30 Which of the following is a Maharatna that is a central public sector enterprise? 

(A) Hindustan Petroleum Corporation Limited 

(B) ONGC Videsh Limited 

(C) NTPC Limited 

(D) PEC Limited 

Answer: (C) 

 

Q. 31 The 2009 Nobel Peace prize was awarded to ………… for his extraordinary efforts to strengthen International Diplomacy. 

(A) Justin Trudeau 

(B) Kofi Annan 

(C) Barack Obama 

(D) Dalai Lama 

Answer: (C) 

 

Q. 32 What is the party symbol of Communist Party of India (M)? 

(A) Elephant 

(B) Lotus flower 

(C) Broom 

(D) Hammer and Sickle 

Answer: (D) 

 

Q. 33 Name the bacteria that causes dysentery. 

(A) Shigella 

(B) Vibrio Cholerae 

(C) Streptococcus pyogenes bacteria 

(D) Mycobacterium tuberculosis 

Answer: (A) 

 

Q. 34 Which of the below is an alloy of mercury? 

(A) Brass 

(B) Nichrome 

(C) Stellite 

(D) Amalgam 

Answer: (D) 

 

Q. 35 Who founded the Swaraj Party? 

(A) C.R. Das and Motilal Nehru 

(B) Bal Gangadhar Tilak and Lala Lajpat Rai 

(C) Mahatma Gandhi and B.R. Ambedkar 

(D) Jawaharlal Nehru and Motilal Nehru 

Answer: (A) 

 

Q. 36 When did the Indian Constituent assembly adopt the constitution? 

(A) 26 November 1949 

(B) 26 January 1950 

(C) 26 November 1950 

(D) 26 January 1949 

Answer: (A) 

 

Q. 37 Liquefied petroleum gas consists of: 

(A) Hexane and benzene 

(B) Nitrogen and ethane 

(C) Propane and butane 

(D) Nonane and octane 

Answer: (C) 

 

Q. 38 Who led the revolt of 1857 in the city of Kanpur? 

(A) Rani Lakshmi Bai 

(B) Nana Sahib 

(C) Begum Hazrat Mahal 

(D) Kunwar Singh 

Answer: (B) 

 

Q. 39 Which of the following is NOT the role of Food Corporation of India? 

(A) Ensuring that farmers are not forced to sell their produce below MSP 

(B) Making laws related to agricultural credit and indebtedness 

(C) Procurement of buffer stock 

(D) Setting up purchase centres to buy produce from the farmers 

Answer: (B) 

 

Q. 40 Who is the winner of Man Booker Award 2018 for the book Milkman? 

(A) Anna Burns 

(B) Daisy Johnson 

(C) Richard Powers 

(D) Robin Robertson 

Answer: (A) 

 

Q. 41 Which Chinese business tycoon(also the richest man of China) recently stepped down from Alibaba, the company he co-founded? 

(A) Jack Ma 

(B) Lei Jun 

(C) Daniel Zhang 

(D) Guo Guangchang 

Answer: (A) 

 

Q. 42 Which major cities of the country are connected by the Golden quadrilateral super highways? 

(A) Delhi-Varanasi-Chennai-Pune-Delhi 

(B) Delhi-Dhanbad-Vishakapatnam-Surat-Delhi 

(C) Delhi-Kolkata-Chennai-Mumbai-Delhi 

(D) Delhi-Kolkata-Angola-Mumbai-Delhi 

Answer: (C) 

 

Q. 43 The new year celebrated by Maharashtra Hindus is referred to by which name? 

(A) Ugadi 

(B) Puthando 

(C) Vishu 

(D) Gudi Padwa 

Answer: (D) 

 

Q. 44 What does road density indicate? 

(A) The length of road per 100 sq. km of land area 

(B) The length of road per 1000 km of land area 

(C) The length of road per 1000 sq. km of land area 

(D) The length of road per 100 km of land area 

Answer: (A) 

 

Q. 45 Out of the given options, which crop grows in black soil? 

(A) Cashewnut 

(B) Sugarcane 

(C) Tea 

(D) Cotton 

Answer: (D) 

 

Q. 46 Pandavani is a folk singing style popular in the state of: 

(A) Bihar 

(B) Jharkhand 

(C) Chhattisgarh 

(D) Uttar Pradesh 

Answer: (C) 

 

Q. 47 The southern most Jyotirlinga temple is located in which Indian state? 

(A) Andhra Pradesh 

(B) Karnataka 

(C) Tamil Nadu 

(D) Kerala 

Answer: (C) 

 

Q. 48 Setting up of which of these is associated with White Revolution in India? 

(A) Mother Dairy 

(B) Paras 

(C) Saras 

(D) Amul 

Answer: (D) 

 

Q. 49 ………. is a chemical process or a reaction between fuel (hydrocarbon) and oxygen. producing heat and light energy resulting in a flame. 

(A) Convection 

(B) Carbonisation 

(C) Combustion 

(D) Conduction 

Answer: (C) 

 

Q. 50 Who was awarded the ‘FIFA Player of the Year 2018’? 

(A) Mohammed Salah 

(B) Christiano Ronaldo 

(C) Luka Modric 

(D) Lionel Messi 

Answer: (C) 

Quant 

Instructions 

For the following questions answer them individually 

Q. 51 The value of 

is equal to 7 + k, where k = 

(A) ⅗ 

(B) ⅕ 

(C) 1/20 

(D) 19/20 

Answer: (D) 

 

Q. 52 A can complete 33⅓% of a work in 5 days and B can complete 40% of the same work in 10 days. They work together for 5 days and then B left the work. A alone will complete the remaining work in: 

(A) 7½ days 

(B) 5 days 

(C) 7 days 

(D) 5½ days 

Answer: (C) 

 

Q. 53 The value of [5.7÷1.9of0.4]÷1.5 / 1.5÷[12.5−(7.5−2.5)] is: 

(A) 0.25 

(B) 0.5 

(C) 25 

(D)

Answer: (C) 

 

Q. 54 If A is 75% more than B, C is 40% less than D and D is 20% more than A, then which one of the following is true? 

(A) 63C = 50B 

(B) 25B = 21C 

(C) 21B = 25C 

(D) 50C = 63B 

Answer: (D) 

 

Q. 55 The average weight of all the students in a class was 45.5 kg. Later on 5 students weighing 44.6 kg 43.4 kg, 44 kg, 45.5 kg and 43.5 kg leave the class. As a result, the average weight of the remaining students increases by 500g. What was the number of students in the class, initially? 

(A) 18 

(B) 20 

(C) 16 

(D) 21 

Answer: (A) 

 

Q. 56 In finding the HCF of two numbers by division method,the last divisor is 174 and the quotient are 1, 7 and 2, respectively. If the sum of the two numbersis divided by. the remainder is: 

(A)

(B)

(C)

(D)

Answer: (A) 

 

Q. 57 A person sells an article at a certain price. What is his gain percent, if by selling it at 80% of that price, he loses 16%? 

(A)

(B) 4.5 

(C)

(D)

Answer: (C) 

 

Q. 58 Study the following table and answer questions: 

Number of candidates appeared in an examination from five cities: 

The number of candidates appearing for the examination from city D is what percent of the total number of candidates failed in cities A and B? 

(A) 72 

(B) 87 

(C) 80 

(D) 90 

Answer: (D) 

 

Q. 59 A shopkeeper allows 20% discount on the marked price of an article and still makes a profit of 28%. If he sells it at 12.5% discount on the marked price, then his gain percent is: 

(A) 20.5 

(B) 24.6 

(C) 36 

(D) 40 

Answer: (D) 

 

Q. 60 A boat goes 2 km upstream and 3 km downstream in 45 minutes while it goes 3 km upstream and 9 km downstream in 90 minutes. What is the speed (in km/hr) of boat when going down stream? 

(A)

(B) 10.5 

(C) 12 

(D) 9.5 

Answer: (C) 

 

Q. 61 What is the compound interest on a sum of ₹7500 for 2¾ years at 8% p.a., the interest being compounded yearly? (nearest to an integer) 

(A) ₹1248 

(B) ₹1773 

(C) ₹1348 

(D) ₹1783 

Answer: (B) 

 

Q. 62 If x is the fourth proportional to 36, 48 and 15, and 4.2, y, 9 and 13.5 are in proportion, then the difference between y and x is: 

(A) 13.7 

(B) 16.4 

(C) 15.6 

(D) 14.8 

Answer: (A) 

 

Q. 63 The driver of a mini bus driving at 82 km/hr locates a motor bike 150 m ahead of him. After 20 seconds, the motor bike was 100 m behind the bus. The speed (in km/hr) of the motor bike is: 

(A) 39 

(B) 37 

(C) 35 

(D) 40 

Answer: (B) 

 

Q. 64 9 men and 12 women can complete a work in 4 days, whereas 3 men and 6 women can complete it in 10 days. The number of days in which 15 women will complete the work is: 

(A)

(B) 10 

(C) 12 

(D)

Answer: (D) 

 

Q. 65 The average of 13 numbers is 65. The average of first three numbers is 63 and that of next numbers is 64.5. The 11th number is 2 more than the 12th number but less than the 13th by 2.5. What is the average of 11th and 12th number? 

(A) 67.5 

(B) 67 

(C) 65.5 

(D) 65 

Answer: (B) 

 

Q. 66 An alloy X contains by weight copper and tin in the ration 7 : 3 and alloy Y contain copper and tin in the ratio 3 : 2. X and Y are taken in the ratio 4 : 1 and melted to form alloy Z. The percentage of copper in alloy Z is: 

(A) 70 

(B) 56 

(C) 65 

(D) 68 

Answer: (D) 

Explanation: 

 

Q. 67 The radius of the base of a conical tent is 5 m and its height is 12 m. how many metres of canvas of width 5 metres will be required to  make it?(use π = 22/7) 

Answer: (B) 

 

Q. 68 The area of a circular park is 37 times the area of a triangular field with sides 20 m, 20 m and 24 m. What is the perimeter (nearest to an integer) of the circular park? 

(A) 300 

(B) 150 

(C) 290 

(D) 350 

Answer: (A) 

 

Q. 69 Study the following table and answer questions: 

Number of candidates appeared in an examination from five cities:

The number of cities, in which pass percentage of candidates is more than 55%,is: 

(A)

(B)

(C)

(D)

Answer: (A) 

 

Q. 70 The median of the observations 10, 11, 13, 17, x + 2, x + 4, 31, 33, 36, 42, arranged in ascending order 24. What is the mean of the data? 

(A) 25.2 

(B) 24.1 

(C) 25.4 

(D) 23.9 

Answer: (B) 

 

Q. 71 A certain sum amounts to ₹11760 in 2½ years at 9% p.a. simple interest. What will be the simple interest on the same sum for 4⅖ years at 15% p.a? 

(A) ₹6336 

(B) ₹6363 

(C) ₹6436 

(D) ₹6433 

Answer: (A) 

 

Q. 72 Water in a canal 3 m wide and 1.2 m deep,is flowing with the velocity of 10 km/h. how much area (in hectares) will it irrigate in half an hour, if 9 cm of standing water is needed? 

(A) 20 

(B) 18.8 

(C) 21 

(D) 20.4 

Answer: (A) 

 

Q. 73 Study the following table and answer questions: 

Number of candidates appeared in an examination from five cities: 

The ratio of the total number of candidates who passed from cities A and C to that who failed in cities B and E is: 

(A) 7 : 9 

(B) 53 : 66 

(C) 27 : 35 

(D) 4 : 7 

Answer: (C) 

 

Q. 74 Ritu sold an article at a loss of 12%. Had she sold it for ₹55.80 more, she would have gained 13%. To gain 10%, she must sell it for ₹: 

(A) 274.80 

(B) 280.46 

(C) 245.52 

(D) 271.60 

Answer: (C) 

 

Q. 75 A sum of 3392 is divided among A, B, C and D such that the ratio for shares of A and B is 2: 5, that of B and C is 3 : 7 and that of C and D is 5 : 8. What is the share of C? 

(A) ₹1060 

(B) ₹1696 

(C) ₹848 

(D) ₹840 

Answer: (A) 

English 

Instructions 

For the following questions answer them individually 

Q. 76 Select the most appropriate word to fill in the blank. 

Fortunately, only a few ………… were hurt in the train accident. 

(A) customers 

(B) passengers 

(C) clients 

(D) vendors 

Answer: (B) 

 

Q. 77 Select the option that means the same as the given idiom. 

State-of-the-art 

(A) Very large 

(B) Scientific 

(C) Creatively made 

(D) Most advanced 

Answer: (D) 

 

Q. 78 Select the word which means the same as the group of words given. 

One who does not drink alcohol. 

(A) vegetarian 

(B) faithful 

(C) religious 

(D) teetotaler 

Answer: (D) 

 

Q. 79 Select the antonym of the given word. 

MAJOR 

(A) Significant 

(B) Leading 

(C) Powerless 

(D) Minor 

Answer: (D) 

 

Q. 80 Select the most appropriate option to fill in the blank. 

I wish to travel the whole world, but I ………… afford it. 

(A) shouldn’t 

(B) wouldn’t 

(C) can’t 

(D) won’t 

Answer: (C) 

 

Q. 81 From the given options, identify the segment in the sentence which contains the grammatical error. I know the most easiest way to solve this problem. 

(A) I know 

(B) most easiest 

(C) to solve 

(D) this problem 

Answer: (B) 

 

Q. 82 Select the synonym of the given word. 

RARELY 

(A) Usually 

(B) Often 

(C) Seldom 

(D) Frequently 

Answer: (C) 

 

Q. 83 Select the synonym of the given word. 

ANNUAL 

(A) Timely 

(B) Routine 

(C) Yearly 

(D) Basic 

Answer: (C) 

 

Q. 84 Select the correctly spelt word. 

(A) Eveluasion 

(B) Evaluation 

(C) Evaluasion 

(D) Evaluetion 

Answer: (B) 

 

Q. 85 Select the most appropriate option to substitute the underlined segment in the given sentence. If there is no need to substitute it, select No improvement. 

The Constitution of India was prepared after a thorough study of constitutions of various countries. 

(A) with a thorough 

(B) before a thorough 

(C) in a thorough 

(D) No improvement 

Answer: (D) 

 

Q. 86 Select the most appropriate option to fill in the blank. 

………….. you hurry up, you will be late. 

(A) Unless 

(B) Although 

(C) If 

(D) Since 

Answer: (A) 

 

Q. 87 Select the most appropriate option to substitute the underlined segment in the given sentence.If there is no need to substitute it, select No improvement. 

Much of the people in the train were asleep at the time of the accident. 

(A) Little of the people 

(B) No improvement 

(C) Most of the people 

(D) More of the people 

Answer: (C) 

 

Q. 88 Select the most appropriate word to fill in the blank. 

He is six feet ……… 

(A) long 

(B) high 

(C) tall 

(D) short 

Answer: (C) 

 

Q. 89 Select the most appropriate option to substitute the underlined segment in the given sentence. If there is no need to substitute it, select No improvement. 

The terrain of India is make of mountains, plateaus and plains. 

(A) No improvement 

(B) is made of 

(C) is making of 

(D) is makes of 

Answer: (B) 

Instructions 

In the following passage some words have been deleted. Fill in the blanks with the help of the alternatives given. 

Passage: 

The sky was always a deep sapphire blue. The snow washard and only by cutting steps in ………..(1) we were able towalk. (A) false move would have been …………..(2) . Stopping at every step, ………….(3) on our axes, wetried to …………..(4) down our hearts, which were ……….. (5) as though they would burst. 

 

Q. 90 Select the most appropriate option that will fill in the blank number 1. 

(A) this 

(B) that 

(C) them 

(D) it 

Answer: (D) 

 

Q. 91 Select the most appropriate option that will fill in the blank number 2. 

(A) danger 

(B) fatal 

(C) disaster 

(D) final 

Answer: (B) 

 

Q. 92 Select the most appropriate option that will fill in the blank number 3. 

(A) leans 

(B) leaned 

(C) leaning 

(D) lean 

Answer: (C) 

 

Q. 93 Select the most appropriate option that will fill in the blank number 4. 

(A) compose 

(B) calm 

(C) pacify 

(D) soothe 

Answer: (B) 

 

Q. 94 Select the most appropriate option that will fill in the blank number 5. 

(A) thumping 

(B) crushing 

(C) thrashing 

(D) moving 

Answer: (A) 

Instructions 

For the following questions answer them individually 

 

Q. 95 Select the word which means the same as the group of words given. 

One who cannot read or write. 

(A) ignoble 

(B) illegible 

(C) illiterate 

(D) ignorant 

Answer: (C) 

 

Q. 96 Select the antonym of the given word. 

URBAN 

(A) Developed 

(B) Rural 

(C) Modern 

(D) Backward 

Answer: (B) 

 

Q. 97 From the given options, identify the segment in the sentence which contains the grammatical error. He is a young man about twenty-five years but very learned. 

(A) He is 

(B) a young man 

(C) but very learned. 

(D) about twenty-five years 

Answer: (D) 

 

Q. 98 Select the option that means the same as the given idiom. 

To keep one’s fingers crossed 

(A) To wish for the best 

(B) To be very happy 

(C) To try very hard 

(D) To be frightened 

Answer: (A) 

 

Q. 99 Select the correctly spelt word. 

(A) Clarence 

(B) Clearance 

(C) Clearence 

(D) Cleerance 

Answer: (B) 

 

Q. 100 From the given options, identify the segment in the sentence which contains the grammatical error. My sister tried to tempt me with sweets so that I ought not tell our parents about the vase she broke. 

(A) with sweets 

(B) ought not tell 

(C) tried to tempt me 

(D) about the vase 

Answer: (B) 

SSC GD 15 Feb 2019 Shift-II Previous Year Paper

SSC GD 15th Feb 2019 Shift-II 

Reasoning 

Instructions 

For the following questions answer them individually 

Q. 1 In a certain code, LOVE is written as OLEV. How will IRIS be written in that code? 

(A) IIRS 

(B) IRSI 

(C) RSII 

(D) RISI 

Answer: (D) 

Explanation: 

LOVE is written as OLEV 

The first word is divided into two parts, and both the letters in both the parts are swapped. ≡ ≡ 

Eg :- LOVE LO and VE, now letters in both parts are swapped = OL and EV OLEV Similarly, IRIS : RISI 

=> Ans – (D) 

 

Q. 2 In a certain code, AXE is written as 120. How will FEE be written in that code? 

(A) 150 

(B) 80 

(C) 60 

(D) 20 

Answer: (A) 

Explanation: 

If all the alphabets in the English alphabetical order are numbered sequentially, i.e. A=1, B=2, C=3 and so on. ≡ 1 × 24 × 5 = 120 

Thus, AXE 

≡ 6 × 5 × 5 = 150 

FEE 

=> Ans – (A) 

 

Q. 3 Choose the odd one out of the given options. 

(A) BFJ 

(B) DGJ 

(C) EGI 

(D) CHI 

Answer: (D) 

Explanation: 

A) : B (+4) = F (+4) = J 

B) : D (+3) = G (+3) = J 

C) : E (+2) = G (+2) = I 

D) : C (+5) = H (+1) = I 

=> Ans – (D) 

 

Q. 4 A square transparent sheet with a pattern is given. How will the pattern appear when the transparent sheet is folded along the dotted line? 

Answer: (A) 

 

Q. 5 Select the option that is related to the third term in the same wayas the second term is related to the first term. 

EFG : JLN :: GKM : ? 

(A) NZY 

(B) LQT 

(C) NUZ 

(D) LQS 

Answer: (B) 

Explanation: 

Expression = EFG : JLN :: GKM : ? 

The pattern followed is : 

Similarly, for GKM : 

=> Ans – (B) 

 

Q. 6 Select the Venn diagram that best represents the given set of classes. 

Furniture, Chairs, Trees 

Answer: (A) 

Explanation: 

All the chairs are types of furniture, but trees are different, hence the venn diagram that best describe above relation is : 

=> Ans – (A) 

 

Q. 7 Eight friends A, B, C, D, E, F, G and H are sitting around circular table facing each other for a lunch. A is opposite D and third to the right of B. G is between A and F. is to the immediate right of A. E is between C and D. Who is sitting opposite C? 

(A)

(B)

(C)

(D)

Answer: (B) 

Explanation: 

A is opposite D and third to the right of B. Also, E is between C and D. 

=> (B) is to the immediate right of D. 

G is between (A) and F, => G is to the immediate left of A. 

H is to the immediate right of A. 

Thus, F is sitting opposite C. 

=> Ans – (B) 

 

Q. 8 Select the option that will correctly replace the Q. mark (?) in the given pattern. 

(A) 206 

(B) 216 

(C)

(D) 36 

Answer: (B) 

 

Q. 9 J, K L, M, N and O are six teachers. Each one teaches a different subject out of Hindi, English, Math, Science. Social Science and Arts, not necessarily in the same order. Each of them teaches on only one day, from Mondayto Saturday. not necessarily in the same order. J teaches Science on Wednesday. O teaches Math second after J. K teaches on the first day of the week, but teaches neither Hindi nor English. M teaches English before N and L teaches Arts before J. Who teaches on Saturday? 

(A)

(B)

(C)

(D)

Answer: (B) 

 

Q. 10 Select a figure from amongst the four alternatives that when placed in the blank space (?) of figure X will complete the pattern. (Rotation is not allowed). 

Answer: (D) 

 

Q. 11 Choose the option that most closely resembles the mirror image of the given figure when the mirror is placed at right side.

Answer: (D) 

Explanation: 

Image in first option is the exact replica of Q. figure, hence it is eliminated. Now, the vertical shape of circular will remain intact, hence second and third options are also not possible. Thus, last option is the correct mirror image. 

=> Ans – (D) 

 

Q. 12 Seven friends O, P, Q, R, S, T and U are watching a movie sitting in a row. P is sitting at one extreme end. Q is sitting to the immediate left of S. P is sitting second to the right of T. U is not sitting at any extreme end. O is sitting between R and T. Who is sitting between P and T? 

(A)

(B)

(C)

(D)

Answer: (C) 

 

Q. 13 Choose the option that would follow next in the given figure series. 

Answer: (B) 

Explanation: 

The pattern followed is that the little gap between the two triangles in the first figure is pointing at top left and is moving in anti clockwise direction, i.e. in the second figure is pointing at left side, then bottom left and then at bottom, thus in the missing figure, it will face bottom right as shown in second figure. 

=> Ans – (B) 

 

Q. 14 Select the option that is related to the third term in the same wayas the second term is related to the first term. 

Pathology : Diseases :: Astronomy : ? 

(A) Future 

(B) Soil 

(C) Planets 

(D) Shells 

Answer: (C) 

 

Q. 15 Select the option that will correctly replace the Q. mark (?) in the series. 

109, 105, 89, 53, ? 

(A) 10 

(B) 11 

(C) -10 

(D) -11 

Answer: (D) 

 

Q. 16 Choose the option in which the figure marked ‘x’ is embedded. (Rotation is not allowed) 

Answer: (D) 

 

Q. 17 Select the option that will correctly replace the Q. mark (?) in the series. 

TOD, RQB, PSZ, NUX,? 

(A) LWW 

(B) LWV 

(C) LVW 

(D) LWU 

Answer: (B) 

Explanation: 

Series : TOD, RQB, PSZ, NUX,? 

The pattern followed in each letter of the terms is : 

1st letter : T (-2) = R (-2) = P (-2) = N (-2) = L 

2nd letter : O (+2) = Q (+2) = S (+2) = U (+2) = W 

3rd letter : (D) (-2) = (B) (-2) = Z (-2) = X (-2) = V 

Thus, missing term : LWV 

=> Ans – (B) 

 

Q. 18 Choose the odd number out of the given options. 

(A) 45 

(B) 75 

(C) 85 

(D) 105 

Answer: (C) 

Explanation: 

Apart from 85, all numbers are divisible by 15, hence it is the odd one. 

=> Ans – (C) 

 

Q. 19 The statements below are followed by two conclusions labeled I and II. Assuming that the information in the statements is true, even if it appears at variance with generally established facts, decide which conclusion(s) logically and definitely follow(s) from the information given in the statements. 

Statements: 

1) All nails are pins. 

2) All hammers are pins. 

Conclusion: 

I. No nail is a pin. 

II. Some pins are nails. 

(A) Neither conclusion I nor conclusion II follows. 

(B) Only conclusion II follows. 

(C) Only conclusion I follows. 

(D) Both conclusions follow: 

Answer: (B) 

 

Q. 20 Select the option that will correctly replace the Q. mark (?) in the series. 

9, 21, 38, 65, ? 

(A) 125 

(B) 115 

(C) 105 

(D) 107 

Answer: (D) 

Explanation: 

The pattern followed is that numbers with successive multiples of 5 between them are added. 9 + 12 = 21 

21 + 17 = 38 

38 + 27 = 65 

65 + 42 = 107 

=> Ans – (D) 

 

Q. 21 Choose the odd one out of the given options. 

(A) Purple 

(B) Yellow 

(C) Red 

(D) Blue 

Answer: (A) 

Explanation: 

Except purple, all the colours appear in a rainbow, hence it is the odd one. 

=> Ans – (A) 

 

Q. 22 Select the option that is related to the third number in the same way as the second number is related to the first number. 

24 : 4 :: 45 : ? 

(A)

(B)

(C)

(D)

Answer: (A) 

 

Q. 23 Select the option that is related to the third term in the same way as the second term is related to the first term. 

Famous : Obscure :: Crazy : ? 

(A) Sane 

(B) Clever 

(C) Sink 

(D) Foolish 

Answer: (A) 

Explanation: 

Obscure means someone who is not discovered or not known, while a famous person is known everywhere, thus the relation between the two words are antonyms, similarly, opposite of crazy is sane. 

=> Ans – (A) 

 

Q. 24 Find out the two signs to be interchanged to make the following equation correct. 

15 + 15 × 15 − 15 ÷ 15 = 15 

(A) − and ÷ 

(B) + and − 

(C) × and ÷ 

(D) + and × 

Answer: (A) 

 

Q. 25 The statements below are followed by two conclusions labeled I and II. Assuming that the information in the statements is true, even if it appears at variance with generally established facts. decide which conclusion(s) logically and definitely follow(s) from the information given in the statements. 

Statements: 

1) Some angers are griefs. 

2) Some griefs are loves. 

Conclusion: 

I. Some griefs are angers. 

II. Some loves are griefs. 

(A) Both conclusions follow. 

(B) Only conclusion II follows. 

(C) Only conclusion I follows. 

(D) Neither conclusion I nor conclusion II follows. 

Answer: (A) 

General knowledge 

Instructions 

For the following questions answer them individually 

Q. 26 With which Mughal ruler is the Moti Masjid in Agra associated? 

(A) Aurangzeb 

(B) Akbar 

(C) Humayun 

(D) Shah Jahan 

Answer: (D) 

 

Q. 27 Which of the following statements is NOT correct about the factors that gave rise to the Consumer Movement in India? 

(A) Frequent food shortages 

(B) Hoarding and Black Marketing 

(C) Adulteration of food 

(D) Tax on soft beverages 

Answer: (D) 

 

Q. 28 Which festival is celebrated to mark the birth anniversary of Prophet Muhammad? 

(A) Eid-ul-fitr 

(B) Shab-i-baraat 

(C) Id-ul-azha 

(D) Milad-un-nabi 

Answer: (D) 

 

Q. 29 What are dolphins, porpoises and whales called? 

(A) Smack 

(B) Amphibian 

(C) Odontocetes 

(D) Cetaceans 

Answer: (D) 

 

Q. 30 Which food delivery service provider was recently acquired by Ola? 

(A) Foodpanda 

(B) Uber eats 

(C) Zomato 

(D) Swiggy 

Answer: (A) 

 

Q. 31 What is the child sex ratio? 

(A) Number of males per thousand females in the age group 0-15 

(B) Number of females per thousand males in the age group 0-6 

(C) Number of females per hundred males in the age group 0-6 

(D) Number of females per hundred males in the age group 0-15 

Answer: (B) 

 

Q. 32 Who won his first Stuttgart Open men’s singles title in June 2018? 

(A) Dominic Thiem 

(B) Nicolas Mahut 

(C) Roger Federer 

(D) Milos Raonic 

Answer: (C) 

 

Q. 33 When was Rowlatt Satyagraha launched and by whom? 

(A) 1929, Mahatma Gandhi 

(B) 1915, Motilal Nehru 

(C) 1919, Mahatma Gandhi 

(D) 1930, C.R. Das 

Answer: (C) 

 

Q. 34 ……… cropping refers to, growing two or more crops simultaneous on the same piece of land 

(A) Simple 

(B) Complex 

(C) Mixed 

(D) Inter 

Answer: (C) 

 

Q. 35 Which of the following is the national aquatic animal of India? 

(A) Whale 

(B) Turtle 

(C) Sea Horse 

(D) River Dolphin 

Answer: (D) 

 

Q. 36 Which of the following is a subject of the state list? 

(A) Banking 

(B) Defence 

(C) Police 

(D) Forests 

Answer: (C) 

 

Q. 37 Which British Viceroy in India carried out the partition of Bengal? 

(A) Lord Canning 

(B) Lord Lytton 

(C) Lord Dalhousie 

(D) Lord Curzon 

Answer: (D) 

 

Q. 38 The National Sports Development Fund (NSDF) wasestablished by the Central Government in the year ……… 

(A) 1997 

(B) 1998 

(C) 1996 

(D) 1999 

Answer: (B) 

 

Q. 39 Which startup was recently honoured by UN for its efforts in cleaning the Ganga river? 

(A) Navdanya Trust 

(B) Satpuda foundation 

(C) HelpUsGreen 

(D) World Wide Fund 

Answer: (C) 

 

Q. 40 ‘Double coincidence of wants’ is a feature of which system? 

(A) Forex system 

(B) Barter system 

(C) Equity system 

(D) Liquidity system 

Answer: (B) 

 

Q. 41 Curd is formed with the help of …………. bacteria. 

(A) Bifidobacterium bifidum 

(B) Lactic acid 

(C) Lactobacillus acidophilus 

(D) Bacillus coagulans 

Answer: (C) 

 

Q. 42 Which of these countries is an example of ‘holding together federation’? 

(A) New Zealand 

(B) Australia 

(C) USA 

(D) India 

Answer: (D) 

 

Q. 43 Kalaripayattu is the martial art form prevalent in which Indian state? 

(A) Tamil Nadu 

(B) Kerala 

(C) Karnataka 

(D) Odisha 

Answer: (B) 

 

Q. 44 Which option represents the crops that are grown in June-July and are harvested in September-October? 

(A) Rice, cotton 

(B) Melon,barley 

(C) Cucumber, watermelon 

(D) Peas, wheat 

Answer: (A) 

 

Q. 45 Which Indian state is the largest bauxite producing state? 

(A) Odisha 

(B) Bihar 

(C) Madhya Pradesh 

(D) Jharkhand 

Answer: (A) 

 

Q. 46 Which tennis player received the Padma Shri in sports in 2018? 

(A) Yuki Bhambri 

(B) Sania Mirza 

(C) Leander Paes 

(D) Somdev Devvarman 

Answer: (D) 

 

Q. 47 Who is the author of the book ‘Exam Warriors’? 

(A) Narendra Modi 

(B) Manmohan Singh 

(C) Sushma Swaraj 

(D) Arun Jaitley 

Answer: (A) 

 

Q. 48 Which declaration was signed at UNCED 1992 at Rio De Janeiro, Brazil? 

(A) Agenda 39 

(B) Agenda 19 

(C) Agenda 22 

(D) Agenda 21 

Answer: (D) 

 

Q. 49 What is the range of pH scale? 

(A) 0 to l4 

(B) 1 to 10 

(C) 1 to 7 

(D) 0 to l0 

Answer: (A) 

 

Q. 50 What was the capital of the Mauryan empire? 

(A) Pataliputra 

(B) Vaishali 

(C) Indraprastha 

(D) Kusinagra 

Answer: (A) 

Quant 

Instructions 

For the following questions answer them individually 

Q. 51 A shopkeeper sells an article at a certain price. What is his gain or loss percent, if by selling it at 75% of that price, he loses 10%? 

(A) Gain, 20% 

(B) Gain, 25% 

(C) Loss, 20% 

(D) Loss, 25% 

Answer: (A) 

 

Q. 52 A man can row downstream a distance of 9 km in 2 hours and takes 6 hours while returning to the starting point. What is the speed (in km/h) of the stream? 

(A)

(B) 2.5 

(C) 1.5 

(D)

Answer: (C) 

 

Q. 53 The average weight of a certain number of students in a group is 54 kg. if 12 students of average weight 52 kg join the group, then the average weight of all the students in the group decreases by 750 g. What was the number of students, initially in the group? 

(A) 18 

(B) 24 

(C) 15 

(D) 20 

Answer: (D) 

 

Q. 54 Surbhi can do a piece of work in 24 days. She completed ⅜ of the work and then left it. Amita complete the remaining work in 10 days. Working together, they will complete 125% of the same work in: 

(A) 10 days 

(B) 9 days 

(C) 12 days 

(D) 15 days 

Answer: (C) 

 

Q. 55 Study the following table and answer question. 

Number of students appeared in an examination from five cities: 

The ratio of the total number of students passed in the examination from cities (A) and (D) to that of students failed from cities (B) and E? 

(A) 9 : 16 

(B) 5 : 9 

(C) 13 : 18 

(D) 14 : 19 

Answer: (D) 

 

Q. 56 The ratio of ages of A and B, 4 years ago, was 5 : 7. The ratio of their ages after 4 years from now will be 7 : 9. What will be the ratio of ages of A and B after 12 years from now? 

(A) 8 : 13 

(B) 13 : 17 

(C) 9 : 11 

(D) 15 : 19 

Answer: (C) 

 

Q. 57 The surface area of a cuboidal box is 240 cm2 and the length of its diagonal is 11 cm. What is the sum (in cm)of its length, breadth and depth? 

(A) 21 

(B) 19 

(C) 20 

(D) 17 

Answer: (B) 

 

Q. 58 The area of an equilateral triangle is 63 times the area of a rhombus whose one side measures 13 cm and one diagonal is 10 cm. The length of side of the triangle, in cm, is: 

(A) 366

(B) 246

(C) 246

(D) 126

Answer: (C) 

 

Q. 59 Let x be the least number which when divided by 12, 15, 18, 20 and 27, the remainder in each case is 2, but x is divisible by 23. If x is divided by the sum of its digits then the quotient is: 

(A) 196 

(B) 149 

(C) 193 

(D) 199 

Answer: (A) 

 

Q. 60 Two trains A and B,186 m and 200 m long are running at 64 km/h and 80 km/h, respectively on parallel tracks. If they are running in the opposite directions, then how much time(in seconds) will a person sitting in the first train A take to cross the other train? 

(A) 5.5 

(B)

(C)

(D) 4.5 

Answer: (B) 

 

Q. 61 The ratios of spirit and water in vessels A and B are 4 : 5 and 7: 11, respectively. The contents of A and B are mixed in the ratio 2 : 5. What is the ratio of spirit and water in the resulting solution? 

(A) 22 : 75 

(B) 53 : 75 

(C) 17 : 25 

(D) 13 : 20 

Answer: (C) 

 

Q. 62 The average of 17 numbers is 29. The average of first 9 numbers is 31 and the average of the last 9 numbers is 27. If 9th number is excluded, then what is the average of the remaining numbers? 

(A) 31 

(B) 31.6 

(C) 29.5 

(D) 29 

Answer: (D) 

 

Q. 63 The value of lies between: 

(A) 10.5 and 11.5 

(B) 11.5 and 12.5 

(C) 9.5 and 10.5 

(D) 8.5 and 9.5 

Answer: (C) 

 

Q. 64 The difference between the mean and median of the data 66, 59, 68, 65, 62, 59, 58, 56, 63, 65 is: 

(A) 0.4 

(B)

(C) 0.8 

(D) 0.5 

Answer: (A) 

 

Q. 65 A certain sum amounts to ₹9243.20 in 1½ years at 4% p.a. simple interest. What will be the simple interest on the same sum for 7½ years at 8% per annum? 

(A) ₹5580 

(B) ₹5223 

(C) ₹5232 

(D) ₹5508 

Answer: (C) 

 

Q. 66 A sum of ₹12000 will amount to ₹x at 12% p.a after 2 years, when the interest is compounded 8-monthly. The value of x is (nearest to an integer) 

(A) 15111 

(B) 15083 

(C) 15117 

(D) 15053 

Answer: (C) 

 

Q. 67 The radius of a metallic sphere is 3 cm.it is melted and drawn into a wire of uniform circular section of 0.1 cm. the length of the wire will be (in m): 

(A) 44 

(B) 24 

(C) 36 

(D) 40 

Answer: (C) 

 

Q. 68 Anu sells an article at a profit of 25%. If she had bought it at 25% less and sold it for ₹30.25 less she would have gained 30%. What is the original price (in ₹) of the article? 

(A) 108 

(B) 105.50 

(C) 105.80 

(D) 110 

Answer: (D) 

 

Q. 69 The ratio of the mean proportional between 0.7 and 2.8 and the third proportional to 2.5 and 3.5 is: 

(A) 7 : 2 

(B) 2 : 7 

(C) 7 : 3 

(D) 3 : 7 

Answer: (B) 

 

Q. 70 Study the following table and answer question. 

Number of students appeared in an examination from five cities: 

The total number of students who appeared in the examination from cities C and D is what percent more than the total number of students who failed in cities A, B and E? (nearest to an integer) 

(A) 44 

(B) 30 

(C) 41 

(D) 42 

Answer: (B) 

 

Q. 71 The value of [⅔ ÷ 8/15 of ⅖ + ⅖ of 7½ ÷ ¾] is: 

(A) 1

(B) 1¼ 

(C) 1½ 

(D) ½ 

Answer: (C) 

 

Q. 72 Study the following table and answer question. 

Number of students appeared in an examination from five cities:

The number of cities in which passing percentage of students is below 56% is: 

(A)

(B)

(C)

(D)

Answer: (B) 

 

Q. 73 Two pipes X and Y can fill an empty tank in ‘t’ minutes. If pipe X alone takes 6 minutes more than ‘t’ to fill the tank and Y alone takes 54 minutes more than ‘t’ to fill the tank, then X and Y together will fill the tank in (in minutes): 

(A) 18 

(B) 12 

(C) 27 

(D) 24 

Answer: (A) 

 

Q. 74 The price of oil is increased by 20%. However, the expenditure on it increases by 15%. What is the percentage increase or decrease in the consumption of oil? 

(A) 6¼%, decrease 

(B) 4⅙%, decrease 

(C) 4⅙%, increase 

(D) 6¼%, increase 

Answer: (B) 

 

Q. 75 A person makes a profit of 17% after allowing 10% discount on its marked price. If he gives 14% discount on the marked price, then his profit percent is: 

(A) 11.8 

(B) 14.6 

(C) 10.6 

(D) 23.8 

Answer: (A) 

English 

Instructions 

For the following questions answer them individually 

Q. 76 Select the most appropriate option to substitute the underlined segment in the given sentence. If there is no need to substitute it, select No Improvement. 

They thought she was arrogant, and she was very shy. 

(A) as soon as she was 

(B) so that she was 

(C) No improvement 

(D) whereas she was 

Answer: (D) 

 

Q. 77 From the given options, identify the segment in the sentence which contains the grammatical error. He not only got a scholarship and also a good job. 

(A) and also 

(B) a good job. 

(C) not only 

(D) got a scholarship 

Answer: (A) 

 

Q. 78 Select the most appropriate option to fill in the blank. 

She did not reply ……….. myLetter. 

(A) against 

(B) for 

(C) with 

(D) to 

Answer: (D) 

 

Q. 79 Select the correctly spelt word. 

(A) Fullfil 

(B) Fullfill 

(C) Fulfil 

(D) Foolfil 

Answer: (C) 

 

Q. 80 Select the word which means the same as the group of words given. 

The practice of marrying one person at a time. 

(A) Monogamy 

(B) Monotony 

(C) Polygamy 

(D) Bigamy 

Answer: (A) 

 

Q. 81 Select the antonym of the given word. 

Attack 

(A) Defence 

(B) Secure 

(C) Assent 

(D) Refuse 

Answer: (A) 

 

Q. 82 Select the correctly spelt word. 

(A) Remembrance 

(B) Remembranse 

(C) Remembarence 

(D) Rememberance 

Answer: (A) 

Instructions 

In the following passage some words have been deleted. Fill in the blanks with the help of the alternatives given. 

Passage: 

Flood is a commonnatural disaster. Scientists have often tried to ……….(1) floods. There are a numberofreasonsthat ……….(2) floods, including heavy rain and cloudbursts. Due to these reasons, 

rivers …………….(3) their banks and cover the surrounding land. Sometimes, dams break and cause disastrous floods that ……………(4) to large-scale destruction. Timely safety ……………(5) 

should be taken to save humanlife and property. 

 

Q. 83 Select the most appropriate option that will fill in the blank number 1. 

(A) control 

(B) predict 

(C) prevent 

(D) avoid 

Answer: (B) 

 

Q. 84 Select the most appropriate option that will fill in the blank number 2. 

(A) occur 

(B) arouse 

(C) happen 

(D) cause 

Answer: (D) 

 

Q. 85 Select the most appropriate option that will fill in the blank number 3. 

(A) wash 

(B) drown 

(C) overflow 

(D) reach 

Answer: (C) 

 

Q. 86 Select the most appropriate option that will fill in the blank number 4. 

(A) lead 

(B) led 

(C) leading 

(D) leads 

Answer: (A) 

 

Q. 87 Select the most appropriate option that will fill in the blank number 5. 

(A) measures 

(B) products 

(C) items 

(D) causes 

Answer: (A) 

Instructions 

For the following questions answer them individually 

 

Q. 88 Select the synonym of the given word. 

IDENTICAL 

(A) Perfect 

(B) Same 

(C) Corresponding 

(D) Varied 

Answer: (B) 

 

Q. 89 Select the synonym of the given word. 

GLEE 

(A) Happiness 

(B) Hearty 

(C) Laughter 

(D) Jump 

Answer: (A) 

 

Q. 90 Select the word which means the same as the group of words given. 

That which can be seen through 

(A) Vague 

(B) Opaque 

(C) Foggy 

(D) Transparent 

Answer: (D) 

 

Q. 91 From the given options, identify the segment in the sentence which contains the grammatical error. The T.V. and the computer has been blamed for reduced reading habits amongst children. 

(A) The T.V. and the computer 

(B) has been blamed 

(C) for reduced 

(D) amongst children. 

Answer: (B) 

 

Q. 92 From the given options, identify the segment in the sentence which contains the grammatical error. I am fastly excited and really looking forward to the trip. 

(A) to the trip 

(B) and really 

(C) looking forward 

(D) I am fastly 

Answer: (D) 

 

Q. 93 Select the most appropriate word tofill in the blank. 

He likes to do stunts with his bike. He is a ………. 

(A) daredevil 

(B) busybody 

(C) wind bag 

(D) little bug 

Answer: (A) 

 

Q. 94 Select the most appropriate option to substitute the underlined segment in the given sentence. If there is no need to substitute it, select No Improvement. 

There were not any excuse for not participating in the competition. 

(A) No improvement 

(B) no excuse 

(C) no excuses 

(D) no any excuse 

Answer: (C) 

 

Q. 95 Select the antonym of the given word. 

ABOVE 

(A) Below 

(B) Less 

(C) Down 

(D) Across 

Answer: (A) 

 

Q. 96 Select the option that means the same as the given idiom. 

To measure swords 

(A) To push an opponent 

(B) To argue with someone 

(C) To confess one’s mistake 

(D) To engage in competition 

Answer: (D) 

 

Q. 97 Select the option that means the same as the given idiom. 

Out of order 

(A) Out of place 

(B) In good condition 

(C) Not in working condition 

(D) Out of nowhere 

Answer: (C) 

 

Q. 98 Select the most appropriate option to substitute the underlined segment in the given sentence. If there is no need to substitute it, select No improvement. 

You need to curb your temper or you might offended someone. 

(A) you might be offend 

(B) you might offend 

(C) No improvement 

(D) you might offending 

Answer: (B) 

 

Q. 99 Select the most appropriate option to fill in the blank. 

I told him to avoid fighting, but he won’t ………. 

(A) doing it 

(B) did it 

(C) do it 

(D) does it 

Answer: (C) 

 

Q. 100 Select the most appropriate word to fill in the blank. 

My mother is ………. riser. 

(A) a fast 

(B) an early 

(C) a quick 

(D) an easy 

Answer: (B) 

SSC GD 15 Feb 2019 Shift-I Previous Year Paper

SSC GD 15th Feb 2019 Shift-I 

Reasoning 

Instructions 

For the following questions answer them individually 

Q. 1 Eight friends A, B, C, D, E, F, G and H are sitting around circular table facing each other for a lunch. A is opposite D and third to the right of B. G is between A and F.H is to the right of A. E is between C and D. Who is sitting third to the right of H? 

(A)

(B)

(C)

(D)

Answer: (C) 

 

Q. 2 The statements below are followed by two conclusions labeled I and II. Assuming that the information in the statements is true, even if it appears at variance with generally established facts, decide which conclusion(s) logically and definitely follow(s) from the information given in the statements. 

Statements: 

1) Some rubies are garments. 

2) Some garments are garnets. 

Conclusion: 

I. Some rubies are garnets. 

II. No ruby is a garnet. 

(A) Only conclusion II follows. 

(B) Only conclusion I follows. 

(C) Both conclusions follow. 

(D) Either conclusion I or conclusion II follows. 

Answer: (D) 

 

Q. 3 A square transparent sheet with a pattern is given. How will the pattern appear when the transparent sheet is folded along the dotted line? 

Answer: (A) 

 

Q. 4 Choose the option in which the figure marked ‘X’ is embedded. (Rotation is not allowed) 

Answer: (C) 

 

Q. 5 J, KL, M, N and are six teachers. Each one teaches a different subject out of Hindi, English, Math, Science, Social Science and Arts, not necessarily in the same order. Each of them teaches on only one day, from Monday to Saturday, not necessarily in the same order. J teaches Science on Wednesday. O teaches Math second day after J. K teaches on the first day of the week, but teaches neither Hindi nor English. M teaches English before N and L teaches Arts before J. Which subject is taught between Thursday and Saturday? 

(A) English 

(B) Math 

(C) Social Science 

(D) Science 

Answer: (B) 

 

Q. 6 Choose the option that would follow next in the given figure series. 

Answer: (A) 

 

Q. 7 Select the option that will correctly replace the Q. mark (?) in the series. 

100, 96, 80, 44, ? 

(A) 64 

(B) 20 

(C) -20 

(D) 16 

Answer: (C) 

 

Q. 8 Select the option that is related to the third term in the same way as the second term is related to the first term. 

Artificial : Natural :: Answer : ? 

(A) Question

(B) Reply 

(C) Problem 

(D) Solution 

Answer: (A) 

 

Q. 9 Choose the odd number out of the given options. 

(A) 21 

(B) 63 

(C) 49 

(D) 37 

Answer: (D) 

 

Q. 10 Choose the odd one out of the given options. 

(A) Kidney 

(B) Nose 

(C) Liver 

(D) Lungs 

Answer: (B) 

 

Q. 11 Seven friends O, P, Q, R, S, T and U are watching a movie sitting in a row. P is sitting at one extremeend. Q is sitting to the immediateleft ofS. P is sitting second to the right of T. U is not sitting at any extreme end. is sitting between R and T. Who is sitting at the extreme right end? 

(A)

(B)

(C)

(D)

Answer: (D) 

 

Q. 12 Select the option that is related to the third number in the same way as the second number is related to the first number. 

45 : 405 :: 49 : ? 

(A) 169 

(B) 637 

(C) 144 

(D) 525 

Answer: (B) 

 

Q. 13 Choose the odd one out of the given options. 

(A) PQH 

(B) HJL 

(C) GJM 

(D) ELM 

Answer: (A) 

 

Q. 14 Select the Venn diagram that best represents the given set of classes. 

Words, Synonyms, Antonymous 

Answer: (B) 

 

Q. 15 In a certain code, STEERING is written as HGVVIRMT.How will Q. be written in that code? 

(A) JFVHGILM 

(B) JFVHGRLM 

(C) JEVHGRLM 

(D) JFVHGRIM 

Answer: (B) 

 

Q. 16 Select the option that will correctly replace the Q. mark (?) in the given pattern. 

(A) 16 

(B) 19 

(C)

(D) 11 

Answer: (D) 

 

Q. 17 Select the option that is related to the third term in the same way as the second term is related to the first term. 

Famous : Renowned :: Fierce : ? 

(A) Will 

(B) Fear 

(C) Desire 

(D) Violent 

Answer: (D) 

 

Q. 18 Select the option that will correctly replace the Q. mark (?) in the series. 

3, 15, 32, 59, ? 

(A) 125 

(B) 105 

(C) 101 

(D) 115 

Answer: (C) 

 

Q. 19 The statements below are followed by two conclusions labeled I and II. Assuming that the information in the statements is true, even if it appears at variance with generally established facts, decide which conclusion(s) logically and definitely follow(s) from the information given in the statements. 

Statements: 

1) All mustards are corns. 

2) All corns are olives. 

Conclusion: 

I. All mustards are olives. 

II. Some olives are mustards. 

(A) Only conclusion I follows. 

(B) Both conclusions follow. 

(C) Only conclusion II follows. 

(D) Either conclusion I or conclusion II follows. 

Answer: (B) 

 

Q. 20 Select the option that will correctly replace the Q. mark (?) in the series. 

PEL, NGK, LIJ, JKI, ? 

(A) HMH 

(B) HHM 

(C) MHH 

(D) MMH 

Answer: (A) 

 

Q. 21 Select the option that is related to the third term in the same wayas the second term is related to the first term. 

AC : BF :: JK : ? 

(A) LM 

(B) UT 

(C) TU 

(D) TV 

Answer: (D) 

 

Q. 22 Find out the two signs to be interchanged to make the following equation correct. 

25 + 5 × 7 − 12 ÷ 3 = 26 

(A) + and ÷ 

(B) + and × 

(C) − and ÷ 

(D) + and − 

Answer: (A) 

 

Q. 23 Select a figure from amongst the four alternatives that when placed in the blank space (?) of figure X will complete the pattern. (Rotation is not allowed). 

Answer: (C) 

 

Q. 24 Choose the option that most closely resembles the mirror image of the given figure when mirror is placed at right side. 

Answer: (B) 

 

Q. 25 In a certain code, UP is written as 336. How will ATE be written in that code? 

(A) 20 

(B) 100 

(C) 80 

(D) 60 

Answer: (B) 

General knowledge 

Instructions 

For the following questions answer them individually 

Q. 26 The tomb of Sher Shah Suri is situated in: 

(A) Sikandara 

(B) Burhanpur 

(C) Jabalpur 

(D) Sasaram 

Answer: (D) 

 

Q. 27 Which of the following cities is associated with the Kumbh Mela? 

(A) Meerut 

(B) Ujjain 

(C) Mathura 

(D) Varanasi 

Answer: (B) 

 

Q. 28 Which of the following Hindu shrines is recently declared as a National Heritage by the Government of Pakistan? 

(A) Katas Raj 

(B) Panj Tirath 

(C) Varun dev temple 

(D) Hinglaj Mata temple 

Answer: (B) 

 

Q. 29 Who is the winner of the Women’s Singles Indonesian Masters Badminton tournament 2019? 

(A) Arundhati Pantawane 

(B) Shivani Gadde 

(C) Saina Nehwal 

(D) P.V. Sindhu 

Answer: (C) 

 

Q. 30 Choose the correct group of diseases that are spread by air: 

(A) Cholera, Typhoid, Tuberculosis 

(B) Tuberculosis. Pneumonia. Influenza 

(C) Warts, common cold, cough 

(D) AIDS, Typhoid, Pneumonia 

Answer: (B) 

 

Q. 31 Under which land revenue system, the zamindars were given the rights to transfer their properties subject to the payment of a fixed amount to the Government? 

(A) Permanent Settlement 

(B) Jagirdari Settlement 

(C) Ryotwari Settlement 

(D) Mahalwari Settlement 

Answer: (A) 

 

Q. 32 Which of the following features of the Organised sector is NOT correct? 

(A) The enterprises under this sector are registered with the Government. 

(B) Workers get paid leaves during the holidays. 

(C) Workers under this sector enjoy security and employment. 

(D) The employers are bound to provide leave travel concession to its workers. 

Answer: (D) 

 

Q. 33 In the context of temple architecture, the most important part of the templeis: 

(A) Gopuram 

(B) Mandapam 

(C) Pradakshina Patha 

(D) Garbhagriha 

Answer: (D) 

 

Q. 34 In which of the following protected areas in India the human interference is prohibited? 

(A) National Parks 

(B) Wetlands 

(C) Wildlife sanctuaries 

(D) Biosphere Reserves 

Answer: (A) 

 

Q. 35 The Zanskar, the Hunza and the Nubra are the tributaries of: 

(A) Indus 

(B) Tapi 

(C) Brahmaputra 

(D) Godavari 

Answer: (A) 

 

Q. 36 Which of the following statements is NOT correct about the culture of Maharashtra? 

(A) Its popular danceis called ‘Gotipua’. 

(B) Ganesh Chaturthiis its main festival. 

(C) Its folk theatre is called Tamasha. 

(D) Its traditional saree is called Paithani. 

Answer: (A) 

 

Q. 37 Who among the following is invited as the chief guest at the 2019 Republic Day of India? 

(A) President of South Africa 

(B) President of Sri Lanka 

(C) Prime Minister of Japan 

(D) President of Nepal 

Answer: (A) 

 

Q. 38 The plastics which get deformed easily on heating are known as: 

(A) Thermosetting Plastics 

(B) Thermo Plastics 

(C) Thermo resistant Plastics 

(D) Thermo dynamic Plastics 

Answer: (B) 

 

Q. 39 With reference to the climatic conditions of the Prairies, which of the following statements is correct? 

(A) The local wind “Loo” blowshere. 

(B) The annual rainfall is very heavy. 

(C) There is a North South wind barrier. 

(D) It has the continental type of climate with extreme temperatures. 

Answer: (D) 

 

Q. 40 The dispute between two states can be resolved by the Supreme Court of India under its: 

(A) Appellate Jurisdiction 

(B) Advisory Jurisdiction 

(C) Supervisory Jurisdiction 

(D) Original Jurisdiction 

Answer: (D) 

 

Q. 41 What will happen to the boiling point and freezing point of water when some common salt is added in it? 

(A) Boiling point elevation, freezing point depression 

(B) No change 

(C) Only boiling point increases or decreases 

(D) Boiling point depression, freezing point elevation 

Answer: (A) 

 

Q. 42 Special Economic Zones are being set up by the Government in order to attract: 

(A) Traditional artisans 

(B) Cooperative companies 

(C) Foreign companies 

(D) Local companies 

Answer: (C) 

 

Q. 43 The B.P. Mandal commission was set up to study the reasons of backwardness of the: 

(A) scheduled tribes 

(B) scheduled castes 

(C) rural women and children 

(D) socially and economically backward classes 

Answer: (D) 

 

Q. 44 Which of the lenses used to cure Myopia? 

(A) convex 

(B) concave 

(C) parabolic 

(D) cylindrical 

Answer: (B) 

 

Q. 45 Who among the following rulers is related to “market regulatory measures” in medieval India? 

(A) Alauddin Khilji 

(B) Sher Shah Suri 

(C) Muhammed Bin Tughlaq 

(D) Jahangir 

Answer: (A) 

 

Q. 46 The other name of Per Capita Income is: 

(A) National Income 

(B) Total Income 

(C) Intermediate Income 

(D) Average Income 

Answer: (D) 

 

Q. 47 Under which of the following acts the system of Dyarchy was introduced in India? 

(A) Government of Indian Act 1935 

(B) Government of Indian Act 1909 

(C) Government of Indian Act 1947 

(D) Government of Indian Act 1919 

Answer: (D) 

 

Q. 48 In 2016 Rio Olympics, Indian gymnast Dipa Karmakar attempted the most dangerous vault known as: 

(A) Yurchenko 

(B) Rudi 

(C) Tsukahara 

(D) Produnova 

Answer: (D) 

 

Q. 49 Napthaline ball disappears after some days dueto the process of: 

(A) sublimation 

(B) melting and evaporation 

(C) melting 

(D) evaporation 

Answer: (A) 

 

Q. 50 The recent largest ever coastal defence exercise is conducted by the Indian Navy is named as: 

(A) SEA SERPENT 

(B) SEA HAWK 

(C) SEA SEIZURE 

(D) SEA VIGIL 

Answer: (D) 

Quant 

Instructions 

For the following questions answer them individually 

Q. 51 A sum of ₹ 10000 is lent on simple interest at the rate of 15% per annum. What is the difference between Simple interest for 6 years and the simple interest for 2 years? 

(A) ₹4000 

(B) ₹8000 

(C) ₹6000 

(D) ₹4500 

Answer: (C) 

Explanation: 

Given, Principal = Rs.10000 

Rate of interest = 15% 

Simple Interest for 1 year = 15% of Rs.10000 = Rs.1500 

Then, Simple Interest for 2 years = 2*1500 = Rs.3000 

Simple Interest for 6 years = 6*1500 = Rs.9000 

Therefore, Required difference = Rs.9000-Rs.3000 = Rs.6000. 

 

Q. 52 What is the mean of the given data? 

2, 3, 4, 5, 6, 7, 8, 9 

(A)

(B) 4.5 

(C) 5.5 

(D)

Answer: (C) 

 

Q. 53 A train is moving with a uniform speed. Train crosses a bridge of length 243 meters in 30 seconds and a bridge of length 343 meters in 36 seconds. What is the speed of the train? 

(A) 60 km/hr 

(B) 72 km/hr 

(C) 64 km/hr 

(D) 65 km/hr 

Answer: (A) 

 

Q. 54 The area of a square and a rectangle are equal. The length of the rectangle is greater than the side of square by 9 cm and its breadth is less than the side of square by 6 cm. What will be the perimeter of the rectangle? 

(A) 84 cm 

(B) 74 cm 

(C) 78 cm 

(D) 76 cm 

Answer: (C) 

 

Q. 55 A car covers a certain distance at a speed of 54 km/hr in 7 hours. What should be the speed of car to cover the same distance in 6 hours? 

(A) 72 km/hr 

(B) 56 km/hr 

(C) 63 km/hr 

(D) 54 km/hr 

Answer: (C) 

 

Q. 56 The curved surface area of a cone is 550 cm2 . If the area of its base is 154 cm2 , then what will be the volume of the cone? 

(A) 2464 cm3 

(B) 1836 cm3 

(C) 1232 cm3 

(D) 1472 cm3 

Answer: (C) 

 

Q. 57 The table given below shows the marks obtained by 4 students in 5 subjects. The maximum marks of each subject is 100.

What is the total percent marks of P4? 

(A) 67.2% 

(B) 59.5% 

(C) 62.6% 

(D) 65.8% 

Answer: (C) 

 

Q. 58 What is the value of (14 ÷ 49 − 3 ÷ 7/2 of 5) ? 

(A) 17/35

(B) 4/35

(C) 6/35

(D) 7/35

Answer: (B) 

 

Q. 59 The product of two numbers is 720. If their Least Common Multiple is 360, then what will be the Highest common Factor of these two numbers? 

(A) 60 

(B) 20 

(C) 120 

(D)

Answer: (D) 

Explanation: 

We know that, Product of two numbers Given, Product of two numbers = 720 LCM = 360 

= LCM × HCF 

Then, 

720 = 360 × HCF 

=> HCF = 2 

 

Q. 60 A person buys 25 kg of rice for ₹600 and sells them at a loss equal to the selling price of 5 kg rice. What will be the loss percentage? 

(A) 14.28% 

(B) 16.66% 

(C) 25% 

(D) 20% 

Answer: (B) 

 

Q. 61 Height of a right circular cylinder is 12 cm. If the radius of its base is 21 cm, then what will be the curved surface area of the cylinder? 

(A) 1544 cm2 

(B) 1584 cm2 

(C) 1456 cm2 

(D) 1632 cm2 

Answer: (B) 

 

Q. 62 What is the value of 24 − [48 ÷ {48 × (48 ÷ (24 × 24))}] ? 

(A) 16 

(B) 12 

(C) 18 

(D) 20 

Answer: (B) 

 

Q. 63 If X/3 = Y/5 = Z/9, then what is the ratio of X : Y : Z?

(A) 9 : 5 : 6 

(B) 3 : 5 : 6 

(C) 15 : 9 : 5 

(D) 3 : 5 : 9 

Answer: (D) 

 

Q. 64 Pipe X can fill a tank in 12 hours and pipe Y can fill the same tank in 36 hours. In how many hours both pipe X and pipe Y together can fill one-third part of the tank? 

(A) 9 hours 

(B) 3 hours 

(C) 15 hours 

(D) 27 hours 

Answer: (B) 

 

Q. 65 Average marks of 15 students of class first is 160. Average marks of 10 students of class second is 120. Average marks of 15 students of class third is 180. What will be the average marks of students of all the classes together? 

(A) 157.5 

(B) 162.5 

(C) 155.5 

(D) 167.5 

Answer: (A) 

 

Q. 66 A person purchases 36 articles for ₹360. How many articles must he sold for ₹360, so that the profit is 50%? 

(A) 18 

(B) 36 

(C) 24 

(D) 32 

Answer: (C) 

 

Q. 67 A, B and C invest ₹14000, ₹18000 and ₹24000 respectively to start a business. If the profit at the end of the year is ₹25480, then what is total share of A and B? 

(A) ₹6370 

(B) ₹14560 

(C) ₹17290 

(D) ₹19110 

Answer: (B) 

 

Q. 68 If S is 160% of T, then T will be what percent of S? 

(A) 85% 

(B) 62.50% 

(C) 65.20% 

(D) 70% 

Answer: (B) 

 

Q. 69 The table given below shows the production of rice and the area under rice cultivation of a country for 5 consecutive years Y1, Y2, Y3, Y4 and Y5. 

What is the percentage change in the yield per square meter of Y5 as compared to Y1? 

(A) 33.33% 

(B) 25% 

(C) 20% 

(D) 37.5% 

Answer: (B) 

 

Q. 70 The table given below shows the production of rice and the area under rice cultivation of a country for 5 consecutive years Y1, Y2, Y3, Y4 and Y5. 

What is the per year average production of rice? 

(A) 702000 kg 

(B) 644000 kg 

(C) 686000 kg 

(D) 668000 kg 

Answer: (D) 

 

Q. 71 The ratio of two numbers is 14 : 25. If the difference between them is 264, then which is the smaller of the two numbers? 

(A) 316 

(B) 294 

(C) 336 

(D) 282 

Answer: (C) 

Explanation: 

Let the two numbers be 14x and 25x 

Difference between the numbers = 25x – 14x = 11x 

Given, 11x = 264 => x = 24 

Then, Smallest number 

= 14x = 14 × 24 = 336 

 

Q. 72 What will be the average of 16, 17, 18, ….. upto fifteen terms? 

(A) 23.5 

(B) 23 

(C) 26 

(D) 24 

Answer: (B) 

 

Q. 73 40 persons can repair a bridge in 12 days. If 8 more persons join them, then in how many days bridge can be repaired? 

(A) 11 days 

(B) 10 days 

(C) 9 days 

(D) 8 days 

Answer: (B) 

 

Q. 74 A sum of ₹14000 is lent at compound interest (interest is compounded annually) for 3 years. If the rate of interest is 10%, then what will be the compound interest? 

(A) ₹4,634 

(B) ₹4,645 

(C) ₹4,364 

(D) ₹4,643 

Answer: (A) 

 

Q. 75 The ratio of the marked price and the cost price of a pen is 3 : 2 while the ratio of marked price and selling price is 5 : 4. What is the profit percentage? 

(A) 20% 

(B) 24% 

(C) 25% 

(D) 16% 

Answer: (A) 

Explanation: 

Given MP:CP=3:2 

Multiplying 5 with numerator and denominator 

MP:CP=15:10 

MP:SP=5:4 

Multiplying 3 with numerator and denominator 

MP:SP=15:12 

Therefore MP:CP:SP=15:10:12 

CP=10x SP=12x 

Profit %=((12x-10x)/10x)*100 

=20% 

English 

Instructions 

For the following questions answer them individually 

Q. 76 Select the option that means the same as the given idiom. 

To keep someone at arm’s length. 

(A) To avoid someone 

(B) To be near someone 

(C) To disturb someone 

(D) To love someone 

Answer: (A) 

 

Q. 77 Select the most appropriate word to fill in the blank. 

You must ………. hard to get the best deal. 

(A) navigate 

(B) neglect 

(C) negate 

(D) negotiate 

Answer: (D) 

 

Q. 78 Select the most appropriate option to substitute the underlined segment in the given sentence. If there is no need to substitute it, select No Improvement. 

The work on two projects are already going on which will connect Leh-Ladakh by railway line and air. 

(A) No improvement. 

(B) is already going on 

(C) is already go on 

(D) is already going up 

Answer: (B) 

 

Q. 79 Select the word which means the same as the group of words given. 

One who feeds on human flesh 

(A) Cannibal 

(B) Omnivorous 

(C) Vegan 

(D) Carnivorous 

Answer: (A) 

 

Q. 80 Select the correctly spelt word. 

(A) Apelling 

(B) Appeeling 

(C) Appeling 

(D) Appealing 

Answer: (D) 

 

Q. 81 Select the word which means the same as the group of words given. 

One who makes official examination of accounts 

(A) Controller 

(B) Supervisor 

(C) Auditor 

(D) Officer 

Answer: (C) 

 

Q. 82 Select the synonym of the given word. 

Forsake 

(A) Accept 

(B) Adopt 

(C) Adapt 

(D) Abandon 

Answer: (D) 

 

Q. 83 From the given options, identify the segment in the sentence which contains the grammatical error. I would rather to go home than go to a restaurant for dinner. 

(A) than go to 

(B) I would 

(C) for dinner. 

(D) rather to go 

Answer: (D) 

 

Q. 84 Select the most appropriate word to fill in the blank. 

The adventurer in him cannot ………. the call of the mountains. 

(A) respond 

(B) resist 

(C) restrain 

(D) restore 

Answer: (B) 

 

Q. 85 From the given options, identify the segment in the sentence which contains the grammatical error. It is a fact that you are known by me. 

(A) by me. 

(B) you are known 

(C) a fact 

(D) It is 

Answer: (A) 

 

Q. 86 Select the synonym of the given word. 

Favoured 

(A) Rejected 

(B) Preferred 

(C) Neglected 

(D) Trusted 

Answer: (B) 

 

Q. 87 Select the most appropriate option to substitute the underlined segment in the given sentence.If there is no need to substitute it, select No improvement. 

Even children shrink away from food thinking that they should become overweight. 

(A) No improvement. 

(B) are become 

(C) were become 

(D) might become 

Answer: (D) 

 

Q. 88 Select the most appropriate option to fill in the blank. 

……….. many cooks spoil the broth. 

(A) So 

(B) (A) few 

(C) Too 

(D) Few 

Answer: (C) 

 

Q. 89 Select the correctly spelt word. 

(A) Mischeivous 

(B) Mischieveous 

(C) Mischievious 

(D) Mischievous 

Answer: (D) 

 

Q. 90 Select the most appropriate option to substitute the underlined segment in the given sentence.If there is no need to substitute it, select No improvement. 

You will be needing to complete all the pending work before it becomes unmanageable. 

(A) No improvement 

(B) would be needing 

(C) will be needed 

(D) will need 

Answer: (D) 

 

Q. 91 Select the antonym of the given word. 

Peculiar 

(A) Usual 

(B) Quiet 

(C) Strange 

(D) Sacred 

Answer: (A) 

 

Q. 92 Select the option that means the same as the given idiom. 

To be at home. 

(A) To be at ease 

(B) To be casual 

(C) To be formal 

(D) To be perplexed 

Answer: (A) 

 

Q. 93 From the given options, identify the segment in the sentence which contains the grammatical error. 

Ritesh is best than many other boys in his class. 

(A) than many 

(B) in the class. 

(C) other boys 

(D) Ritesh is best 

Answer: (D) 

Instructions In the following passage some words have been deleted. Fill in the blanks with the help of the alternatives given. 

Passage: 

The Last one in this long procession of silent men and women was a thin old man. Even (1)………. carried a load of two baskets, slung on a pole on his (2)……….. In one basket there were (3)…………. pots. The other basket it seemed there was a quilt, extremely (4)………… and patched, but clean still. (5)………… the load was light, it was too much for the old man. 

Q. 94 Select the most appropriate option that will fill in the blank number 1. 

(A) they 

(B) it 

(C) he 

(D) she 

Answer: (C) 

 

Q. 95 Select the most appropriate option that will fill in the blank number 2. 

(A) head 

(B) neck 

(C) hand 

(D) shoulder 

Answer: (D) 

 

Q. 96 Select the most appropriate option that will fill in the blank number 3. 

(A) one 

(B) some 

(C) any 

(D) none 

Answer: (B) 

 

Q. 97 Select the most appropriate option that will fill in the blank number 4. 

(A) ragged 

(B) beautiful 

(C) shining 

(D) new 

Answer: (A) 

 

Q. 98 Select the most appropriate option that will fill in the blank number 5. 

(A) Although 

(B) Because 

(C) Since 

(D) But 

Answer: (A) 

Instructions 

For the following questions answer them individually 

 

Q. 99 Select the antonym of the given word. 

Ruthless 

(A) Humble 

(B) Merciless 

(C) Careful 

(D) Kind 

Answer: (D) 

 

Q. 100 Select the most appropriate option to fill in the blank. 

All good things ………. come to an end 

(A) must 

(B) ought 

(C) shall not 

(D) have not 

Answer: (A) 

SSC GD 14 Feb 2019 Shift-III Previous Year Paper

SSC GD 14th Feb 2019 Shift-III

Reasoning 

Instructions 

For the following questions answer them individually 

Q. 1 In a certain code, DIG is written as 60. How will FEED be written in that code? 

(A) 90 

(B) 80 

(C) 20 

(D) 50 

Answer: (B) 

Explanation: 

The sum of the numbers represented by each alphabet, as A=1, B=2, C=3 and so on, is multiplied by the number of letters in the word. ≡ (4 + 9 + 7) × 3 = 60 

Eg :- DIG 

≡ (6 + 5 + 5 + 4) × 4 = 80 

Similarly, FEE(D) 

=> Ans – (B) 

 

Q. 2 Eight friends A, B, C, D, E, F, G and H are sitting around circular table facing each other for a lunch. A is opposite F and third to the right of B. G is between F and D.H is to the left of D. E is between C and A. Who is sitting third to the right of H? 

(A)

(B)

(C)

(D)

Answer: (D) 

Explanation: 

A is opposite F and third to the right of B, => B is sitting to the immediate right of F. 

G is between F and D, and H is to the left of D, => H sits opposite B and D to the immediate right of H. 

E is between (C) and A, => (C) sits to the immediate right of B. 

=> F is sitting third to the right of H. 

=> Ans – (D) 

 

Q. 3 A square transparent sheet with a pattern is given. How will the pattern appear when the transparent sheet is folded along the dotted line? 

Answer: (C) 

 

Q. 4 The statements below are followed by two conclusions labeled I and II. Assuming that the information in the statements is true, even if it appears at variance with generally established facts, decide which conclusion(s) logically and definitely follow(s) from the information given in the statements. 

Statements: 

1.) All pens are frogs. 

2.) All crows are frogs. 

Conclusions: 

I. No pen is a crow. 

II. Some pens are crows. 

(A) Only conclusion II follows. 

(B) Both conclusions follow. 

(C) Only conclusion I follows. 

(D) Either conclusion I or conclusion II follows. 

Answer: (D) 

Explanation: 

The venn diagram for above statements is : 

Conclusions: 

I. No pen is a crow = may or may not be true. 

II. Some pens are crows = may or may not be true. 

Thus, either conclusion I or II follows. 

=> Ans – (D) 

 

Q. 5 Select the option that is related to the third term in the same way as the second term is related to the first term. 

BFH : HBF :: GKM : ? 

(A) KMG 

(B) MKI 

(C) MGK 

(D) IKM 

Answer: (C) 

Explanation: 

Expression = BFH : HBF :: GKM : ? 

The pattern followed is that last letter of the first term is appended at the beginning, rest is same. ≡ 

Eg :- BF-H H-BF = HBF 

Similarly, GKM : MGK 

=> Ans – (C) 

 

Q. 6 Select a figure from amongst the four alternatives that when placed in the blank space (?) of figure X will complete the pattern. (Rotation is not allowed). 

Answer: (B) 

Explanation: 

If we complete the above figure, then the missing pattern is represented by the red colour.=> Ans – (B) 

 

Q. 7 Choose the odd number out of the given options. 

(A) 216 

(B) 65 

(C)

(D) 125 

Answer: (B) 

Explanation: 

Apart from 65, all are perfect cubes, hence it is the odd one. 

=> Ans – (B) 

 

Q. 8 Choose the option in which the figure marked ‘x’ is embedded. (Rotation is not allowed) 

Answer: (C) 

 

Q. 9 Seven friends O, P, Q, R, S, T and U are watching a movie sitting in a row. S is sitting at one extreme end. O is sitting second to the right of S. P is sitting between O and Q. U is NOT sitting at any extreme end.R is sitting to the immediate left of T. Who is sitting fourth to the right of U? 

(A)

(B)

(C)

(D)

Answer: (A) 

 

Q. 10 Choose the option that most closely resembles the mirror image of the given word when mirror is placed at right side. 

Answer: (D) 

 

Q. 11 Select the option that is related to the third term in the same way as the second term is related to the first term. 

Dispur : Assam :: Kohima : ? 

(A) Capital 

(B) Sikkim 

(C) Meghalaya 

(D) Nagaland 

Answer: (D) 

Explanation: 

First is the capital of second, Assam’s capital is Dispur, similarly Kohima is the capital of Nagaland

=> Ans – (D) 

 

Q. 12 Select the option that will correctly replace the Question mark (?) in the series. 

6, 21, 51, 96, ? 

(A) 156 

(B) 176 

(C) 136 

(D) 116 

Answer: (A) 

Explanation: 

Consecutive multiples of 15 are added. 

6 + 15 = 21 

21 + 30 = 51 

51 + 45 = 96 

96 + 60 = 156 

=> Ans – (A) 

 

Q. 13 Choose the odd one out of the given options. 

(A) LNM 

(B) RNK 

(C) JKV 

(D) HLW 

Answer: (A) 

Explanation: 

Only in the first option, there are consecutive letters as they appear in English alphabetical order, hence it is the odd one. => Ans – (A) 

 

Q. 14 Find out the two signs to be interchanged to make the following equation correct. 

27 + 48 × 2 − 7 ÷ 3 = 30 

(A) × and ÷ 

(B) + and − 

(C) − and ÷ 

(D) − and × 

Answer: (A) 

 

Q. 15 Choose the option that would follow next in the given figure series. 

Answer: (B) 

 

Q. 16 Select the option that will correctly replace the Question mark (?) in the series. 

TOD, RQB, PSZ, NUX, ? 

(A) LWU 

(B) LVW 

(C) LWW 

(D) LWV 

Answer: (D) 

Explanation: 

Expression : TOD, RQB, PSZ, NUX, ? 

The pattern followed in each letter of the terms is : 

1st letter : T (-2) = R (-2) = P (-2) = N (-2) = L 

2nd letter : O (+2) = Q (+2) = S (+2) = U (+2) = W 

3rd letter : (D) (-2) = (B) (-2) = Z (-2) = X (-2) = V 

Thus, missing term = LWV 

=> Ans – (D) 

 

Q. 17 Select the option that is related to the third number in the same way as the second number is related to the first number. 

45 : 315 :: 37 : ? 

(A) 144 

(B) 259 

(C) 169 

(D) 221 

Answer: (B) 

Explanation: 

Expression = 45 : 315 :: 37 : ? 

x : 7x 

The pattern followed is = 

Eg :- 

45 × 7 = 315 

Similarly, 

37 × 7 = 259 

=> Ans – (B) 

 

Q. 18 In a certain code, STEERING is written as TEIGTEIG. How will Q. be written in that code? 

(A) USINVSIN 

(B) USJNUSIN 

(C) USINUSIN 

(D) USINUSJN 

Answer: (C) 

Explanation: 

STEERING is written as TEIGTEIG 

The pattern followed is that the letters at even positions (starting from left) are taken and are written twice. ≡ 

Eg : S(T)E(E)R(I)N(G) TEIG + TEIG = TEIGTEIG 

Similarly, for Q. : USINUSIN 

=> Ans – (C) 

 

Q. 19 The statements below are followed by two conclusions labeled I and II. Assuming that the information in the statements is true, even if it appears at variance with generally established facts, decide which conclusion(s) logically and definitely follow(s) from the information given in the statements. 

Statements: 

1.) Some owls are horses. 

2.) Some horses are hares. 

Conclusions: 

I. Some owls are hares. 

II. Some hares are owls. 

(A) Only conclusion II follows. 

(B) Only conclusion I follows. 

(C) Both conclusions follow. 

(D) Neither conclusion I nor conclusion II follows. 

Answer: (D) 

 

Q. 20 Choose the odd one out of the given options. 

(A) Ghana 

(B) Cuba 

(C) Egypt 

(D) Berlin 

Answer: (D) 

Explanation: 

Ghana, Cuba and Egypt are countries, while Berlin is the capital of Germany, hence it is the odd one. 

=> Ans – (D) 

 

Q. 21 Select the option that will correctly replace the Question mark (?) in the series. 

5, 6, 14, 45, ? 

(A) 135 

(B) 184 

(C) 195 

(D) 180 

Answer: (B) 

 

Q. 22 Select the option that will correctly replace the Question mark (?) in the given pattern. 

(A) 34 

(B) 15 

(C) 64 

(D)

Answer: (C) 

Explanation: 

The pattern followed is : 

(2 + 2)2 = (4)2 = 16 

(2 + 4)2 = (6)2 = 36 

(3 + 5)2 = (8)2 = 64 

=> Ans – (C) 

 

Q. 23 J, K L, M, N and O are six teachers. Each one teaches a different subject out of Hindi, English, Math, Science, Social Science and Arts, NOT necessarily in the same order. Each of them teaches on only one day, from Monday to Saturday, NOT necessarily in the same order. J teaches Arts on Saturday. L teaches neither English nor Social Science, but he teaches on Thursday. Wednesday is reserved for Maths taught by K. O teaches Science a day before N. Social Science is taught a day before Arts. Which subject is taught on the day between Thursday and Saturday? 

(A) Social Science 

(B) Maths 

(C) Science 

(D) English 

Answer: (A) 

 

Q. 24 Select the option that is related to the third term in the same way as the second term is related to the first term. 

Famous : Renowned :: Moisten : ? 

(A) Drench 

(B) Desire 

(C) Debate 

(D) Drink 

Answer: (A) 

Explanation: 

The first two words are pairs of synonyms, similarly word which has same meaning as moisten is drench. => Ans – (A) 

 

Q. 25 Select the Venn diagram that best represents the given set of classes. 

Whales, Bats, Mammals 

Answer: (D) 

General knowledge 

Instructions 

For the following questions answer them individually 

Q. 26 Which of the following Union Territories is given partial statehood in India? 

(A) Chandigarh 

(B) Dadra and Nagar Haveli 

(C) Daman Diu 

(D) Puducherry 

Answer: (D) 

 

Q. 27 Malaria and Kala Azar are caused by: 

(A) bacteria 

(B) fungi 

(C) protozoa 

(D) virus 

Answer: (C) 

 

Q. 28 Choose the correct group of important Gods of the Vedic period. 

(A) Brahma, Prajapati, Garuda 

(B) Seshanaga, Chandra, Yama 

(C) Varuna, Vishnu, Shiva 

(D) Agni, Indra, Soma 

Answer: (D) 

 

Q. 29 Which of the following statements is NOT correct about the Tundra type of vegetation? 

(A) Small shrubsare a part of this vegetation. 

(B) Moss and lichens are found here. 

(C) The growth of natural vegetation is limited here. 

(D) It is found in a temperate region. 

Answer: (D) 

 

Q. 30 The Justice Rajendar Sachar Committee was set up by the Government of India to : 

(A) study the condition of unorganized sector of Indian Economy. 

(B) study the conditions of National parks in India. 

(C) examine the status of India’s foreign policy. 

(D) examine the social and economic status of the Muslim community. 

Answer: (D) 

 

Q. 31 In the context of Elections in India, the term VVPAT stands for: 

(A) Voter Visit Poll Account Trail 

(B) Voter Vivid Press Audit Trail 

(C) Voter Verifiable Paper Audit Trail 

(D) Voter Verifiable Paper Account Trail 

Answer: (C) 

 

Q. 32 Which of the following places served as the venue for the closing ceremony of Pravasi Bharatiya Divas 2019? 

(A) Chandigarh 

(B) Pune 

(C) Varanasi 

(D) Kochi 

Answer: (C) 

 

Q. 33 The other name of the tertiary sector is : 

(A) service sector 

(B) primary sector 

(C) manufacturing sector 

(D) public sector 

Answer: (A) 

 

Q. 34 ‘Bibi Ka Maqbara, the replica of the Taj Mahal is situated in: 

(A) Chandarnagar 

(B) Aurangabad 

(C) Ahmedabad 

(D) Cuttack 

Answer: (B) 

 

Q. 35 In the context of the Indian Constitution, which of the following Articles guarantees the Right to Freedom of Religion to its citizens? 

(A) Article 14 

(B) Article 25 

(C) Article 32 

(D) Article 22 

Answer: (B) 

 

Q. 36 The type of unemployment in the agricultural sector of India where more people are employed than required is known as: 

(A) disguised unemployment 

(B) seasonal unemployment 

(C) structural unemployment 

(D) marginal unemployment 

Answer: (A) 

 

Q. 37 Sandstone is an example of : 

(A) non-foliated rock 

(B) sedimentary rock 

(C) igneous rock 

(D) metamorphic rock 

Answer: (B) 

 

Q. 38 Which bridge connects Dibrugarh in Assam to Pasighat in Arunachal Pradesh? 

(A) Naini 

(B) Bogibeel 

(C) Coronation 

(D) Pamban 

Answer: (B) 

 

Q. 39 Food containing oil and fat, when kept for a longer time, causes a change in its taste and smell due to the process of ………. 

(A) decomposition 

(B) fermentation 

(C) rancidity 

(D) substitution 

Answer: (C) 

 

Q. 40 The Ryotwari System was introduced by: 

(A) Warren Hastings 

(B) Lord Cornwallis 

(C) Thomas Munro 

(D) Lord Ripon 

Answer: (C) 

 

Q. 41 With reference to the Mahatma Gandhi National Rural Employment Guarantee Act, which of the following statements is NOT correct? 

(A) It is also known as the Right to Work. 

(B) It is implemented by the Ministry of Rural Development. 

(C) It was initiated by the Government of India in 2009. 

(D) It guarantees 100 days of work to the beneficiary. 

Answer: (C) 

 

Q. 42 The Bandipur National Park is located in which state of India? 

(A) Karnataka 

(B) Tamil Nadu 

(C) Odisha 

(D) Chhattisgarh 

Answer: (A) 

 

Q. 43 Light Year is related to : 

(A) weather 

(B) distance 

(C) sound 

(D) speed 

Answer: (B) 

 

Q. 44 The ………. collects sound from the surroundings. 

(A) auditory nerve 

(B) tympanic membrane 

(C) cochlea 

(D) pinna 

Answer: (D) 

 

Q. 45 The 2018 Dronacharya Award for Table Tennis was given to : 

(A) Srinivasa Rao 

(B) Jiwan Kumar Sharma 

(C) Vijaya Sharma 

(D) Tarak Sinha 

Answer: (A) 

 

Q. 46 The Lahore session of the Indian National Congress is significant because: 

(A) it passed the resolution to demand Poorna Swaraj. 

(B) it passed the resolution to start the Rowlatt Satyagraha. 

(C) it passed the resolution to hold civil services examination in India. 

(D) it passed the resolution to boycott provincial elections. 

Answer: (A) 

 

Q. 47 After some days, statues of brass and bronze become green due to: 

(A) oxidation 

(B) corrosion 

(C) erosion 

(D) decomposition 

Answer: (B) 

 

Q. 48 The bronze sculpture of Nataraja was cast during the period of the: 

(A) Cheras 

(B) Pallavas 

(C) Cholas 

(D) Pandyas 

Answer: (C) 

 

Q. 49 ‘Bali Yatra’, the voyage to Bali, is celebrated in which of the following states of India? 

(A) Tamil Nadu 

(B) Rajasthan 

(C) Kerala 

(D) Odisha 

Answer: (D) 

 

Q. 50 Which Indian cricketer has become the fastest bowler to claim 100 wickets in One-day Internationals as on January 2019? 

(A) Ishant Sharma 

(B) Umesh Yadav 

(C) Hardik Pandya 

(D) Mohammed Shami 

Answer: (D) 

Quant 

Instructions 

For the following questions answer them individually 

Q. 51 What is the value of [(½ ÷ ¼ – 4 × ½ × ⅓) of (⅔ ÷ 4/3 + ½)]? 

(A) ½ 

(B) 5/2 

(C) ¾ 

(D) 4/3 

Answer: (D) 

 

Q. 52 K alone can complete a work in 20 days and M alone can complete the same work in 30 days. K and M start the work together but K leaves the work after 5 days of the starting of work. In how many days M will complete the remaining work? 

(A) 35/2 days 

(B) 25/2 days 

(C) 55/2 days 

(D) 45/2 days 

Answer: (A) 

 

Q. 53 (A) sum becomes three times of itself in 10 years at the rate of simple interest. What is the rate of interest per annum? 

(A) 25% 

(B) 20% 

(C) 15% 

(D) 30% 

Answer: (B) 

 

Q. 54 The sum of the volume of two solid spheres is 1144/3 cm3. If the sum of their radii is 7 cm, then what will be the difference of the radii? 

(A) 4 cm 

(B) 3 cm 

(C) 2 cm 

(D) 1 cm 

Answer: (D) 

 

Q. 55 Rohit, Mohit and Sumit individualy can complete a work in 8 days, 14 days and 21 days respectively. In how many days these three together can complete the same work? 

(A) 156/43 days 

(B) 168/41 days 

(C) 168/43 days 

(D) 156/41 days 

Answer: (B) 

 

Q. 56 What is the mode of the given data? 

3, 0, 1, 0, 2, 1, 2, 0, 1, 2, 1, 1, 1, 3, 2 

(A)

(B)

(C)

(D)

Answer: (A) 

Explanation: 

Arranging in ascending order : 0, 0, 0, 1, 1, 1, 1, 1, 1, 2, 2, 2, 2, 3, 3 

Thus, the most repeated term = Mode =

=> Ans – (A) 

 

Q. 57 If a number is divided by 14, then it becomes equal to 10% of the second number. First number is how much percent more than the second number? 

(A) 40% 

(B) 45% 

(C) 60% 

(D) 30% 

Answer: (A) 

 

Q. 58 If the selling price of an article is ₹638 and profit is 45%, then what will be the net profit? 

(A) ₹256 

(B) ₹287.1 

(C) ₹440 

(D) ₹198 

Answer: (D) 

 

Q. 59 If Sohan is driving a bike at a speed of 25 m/s, then in how much time Sohan will cover a distance of 540 km? 

(A) 7 hr 

(B) 5 hr 

(C) 21.6 hr 

(D) 6 hr 

Answer: (D) 

 

Q. 60 The area of a square and a rectangle are equal. The length of the rectangle is greater than the side of a square by 5 cm and its breadth is less than the side of square by 4 cm. What will be the perimeter of the rectangle? 

(A) 84 cm 

(B) 82 cm 

(C) 76 cm 

(D) 72 cm 

Answer: (B) 

 

Q. 61 The ratio of two numbers is 13 : 20. If the difference between them is 189, then what is the smaller of the two numbers? 

(A) 279 

(B) 221 

(C) 297 

(D) 351 

Answer: (D) 

 

Q. 62 Marked price of an article is 40% more than its cost price. If 50% discount is given on the article, then selling price is what percent of the cost price? 

(A) 78% 

(B) 70% 

(C) 93% 

(D) 84% 

Answer: (B) 

 

Q. 63 The line chart given below shows the production (in 10000s) of cars in 7 different months. 

What is the average monthly production for the given 7 months? 

(A) 32.2 (in 10000s) 

(B) 21.6 (in 10000s) 

(C) 23.5 (in 10000s) 

(D) 28.8 (in 10000s) 

Answer: (D) 

 

Q. 64 The table given below shows the marks obtained by 4 students in 5 subjects. The maximum marks of each subject is 100.

What are the average marks per subject obtained by P2? 

(A) 72.4 

(B) 75.8 

(C) 70.6 

D74.6 

Answer: (A) 

 

Q. 65 What is the value of [45 of ( 3/7 ÷ 15/14) − 6½ ÷ 3 − 4) + 2 ]? 

(A) 131/6

(B) 147/11

(C) 109/6

(D) 33/7

Answer: (A) 

 

Q. 66 If Mohit can cover a distance of 1872 km in 26 hours, then what is the speed of Mohit? 

(A) 36 km/hr 

(B) 72 km/hr 

(C) 20 m/s 

(D) Both (B) and (C) 

Answer: (D) 

 

Q. 67 A person purchases calculators at the rate of ₹345 per calculator. If he sells calculators on 40% loss then what will be the selling price of 28 calculators? 

(A) ₹6872 

(B) ₹8657 

(C) ₹13524 

(D) ₹5796 

Answer: (D) 

 

Q. 68 Height of a right circular cylinder is 20 cm.If the radius is its base 14 cm, then what will be the curved surface area of the cylinder? 

(A) 1940 cm2 

(B) 2120 cm2 

(C) 1760 cm2 

(D) 1880 cm2 

Answer: (C) 

 

Q. 69 The average salary of managers is x and the average salary of workers is y. The number of the managers is 15 times of the number of workers. What is average salary of managers and workers together? 

(A) (15x+y)/16 

(B) (15x+y)/(x+y) 

(C) (x+15y)/(x+y) 

(D) (x+15y)/16 

Answer: (A) 

 

Q. 70 The average weight of 10 articles is 65 kg. If three articles having an average weight of 72 kg are removed, then what will be new average weight of the remaining articles? 

(A) 58 kg 

(B) 65 kg 

(C) 71 kg 

(D) 62 kg 

Answer: (D) 

 

Q. 71 P, Q and R invest ₹14000, ₹18000 and ₹24000 respectively to start a business. If the profit at the end of the year is ₹25480, then what is difference between the profits of P and Q? 

(A) ₹2670 

(B) ₹3480 

(C) ₹1820 

(D) ₹2140 

Answer: (C) 

 

Q. 72 What will be the highest number which when divides 52 and 104 leaves remainder 4 and 8 respectively? 

(A) 12 

(B) 48 

(C) 16 

(D) 26 

Answer: (B) 

 

Q. 73 If x : y = 5 : 4, then what will be the ratio of (x/y) : (y/x)? 

(A) 25 : 16 

(B) 16 : 25 

(C) 4 : 5 

(D) 5 : 4 

Answer: (A) 

 

Q. 74 The line chart given below shows the production (in 10000s)of cars in 7 different months. 

Production of cars in month M2 is how much percent more than the production of cars in month M1? 

(A) 25% 

(B) 37.5% 

(C) 33.33% 

(D) 28% 

Answer: (D) 

 

Q. 75 A sum of ₹4500 is lent at compound interest. If the rate of interest is 10% per annum (interest is compounded annually), then what will be the amount after 3 years? 

(A) ₹5989.5 

(B) ₹5689.5 

(C) ₹5686.5 

(D) ₹5889.5 

Answer: (A) 

English 

Instructions 

For the following questions answer them individually 

Q. 76 Select the most appropriate option to substitute the underlined segment in the given sentence. If there is no need to substitute it, select No Improvement. 

The prince was told kill the dragon,but in order to do so, he had to cross the Forbidden Mountain. 

(A) was told killing the dragon 

(B) No improvement 

(C) was told too kill the dragon 

(D) was told to kill the dragon 

Answer: (D) 

 

Q. 77 From the given options, identify the segment in the sentence which contains the grammatical error. 

Ten years since Mohit left India and started working for the betterment of the downtrodden people. 

(A) Ten years since 

(B) the betterment of 

(C) started working for 

(D) Mohit left India 

Answer: (A) 

 

Q. 78 Select the option that means the same as the given idiom. 

At the drop of a hat 

(A) Instantly 

(B) Lately 

(C) Slowly 

(D) Seldom 

Answer: (A) 

 

Q. 79 Select the correctly spelt word. 

(A) Calander 

(B) Calendar 

(C) Calandar 

(D) Calender 

Answer: (B) 

 

Q. 80 Select the most appropriate optionto fill in the blank. 

The film we sawlast night was boring. I wish I ………. seen it. 

(A) didn’t 

(B) hadn’t 

(C) shouldn’t 

(D) wouldn’t have 

Answer: (B) 

 

Q. 81 Select the correctly spelt word. 

(A) Mentainance 

(B) Maintainance 

(C) Maintainence 

(D) Maintenance 

Answer: (D) 

 

Q. 82 Select the antonym of the given word. 

BOISTEROUS 

(A) Aggravated 

(B) Incompetent 

(C) Restrained 

(D) Exuberant 

Answer: (C) 

 

Q. 83 Select the word which means the same as the group of words given. 

A person who thinks he/she is the best. 

(A) Eccentric 

(B) Egoist 

(C) Selfish 

(D) Garrulous 

Answer: (B) 

 

Q. 84 Select the most appropriate option to substitute the underlined segment in the given sentence. If there is no need to substitute it, select No Improvement. 

The mobile company changed their handsets last year, and by doing so they have managed to improve their sale. 

(A) No improvement 

(B) improve the sale 

(C) improve their sales 

(D) improved their sales 

Answer: (C) 

 

Q. 85 Select the synonym of the given word. 

AMIABLE 

(A) Cheerful 

(B) Bashful 

(C) Hostile 

(D) Friendly 

Answer: (D) 

 

Q. 86 Select the most appropriate option to substitute the underlined segment in the given sentence.If there is no need to substitute it, select No Improvement. 

Don’t use those pencils; use the one we bought in Portugal. 

(A) No improvement 

(B) use this ones 

(C) use one 

(D) use the ones 

Answer: (D) 

 

Q. 87 Select the synonym of the given word. 

TURBULENT 

(A) Violent 

(B) Miserable 

(C) Accurate 

(D) Troublesome 

Answer: (A) 

 

Q. 88 From the given options, identify the segment in the sentence which contains the grammatical error. IfI ama bird, I would fly and explore the world. 

(A) I would fly 

(B) and explore 

(C) If I am 

(D) the world 

Answer: (C) 

 

Q. 89 Select the most appropriate word to fill in the blank. 

Beware of him. He is as sly as a __________

(A) goat 

(B) rabbit 

(C) snail 

(D) fox 

Answer: (D) 

Instructions 

In the following passage some words have been deleted. Fill in the blanks with the help of the alternatives given. 

Passage: 

Most animals have their own habitats ……….(1) some make long journeys from one region to another. This ……….(2) called migration. It takes place at a particular …………(3) of the year. Birds are the best-known …………(4) but many other animals also migrate to places with …………..(5) climate. 

 

Q. 90 Select the most appropriate option to fill in blank number(1). 

(A) but 

(B) because 

(C) so 

(D) as 

Answer: (A) 

 

Q. 91 Select the most appropriate option to fill in blank number(2). 

(A) are 

(B) were 

(C) is 

(D) was 

Answer: (C) 

 

Q. 92 Select the most appropriate option to fill in blank number(3). 

(A) season 

(B) time 

(C) month 

(D) part 

Answer: (B) 

 

Q. 93 Select the most appropriate option to fill in blank number(4). 

(A) visitors 

(B) migrants 

(C) travellers 

(D) tourists 

Answer: (B) 

 

Q. 94 Select the most appropriate option to fill in blank number(5). 

(A) extreme 

(B) favourable 

(C) harsh 

(D) cold 

Answer: (B) 

Instructions 

For the following questions answer them individually 

 

Q. 95 Select the antonym of the given word. 

URGE 

(A) Deter 

(B) Persuade 

(C) Triumph 

(D) Request 

Answer: (A) 

 

Q. 96 Select the most appropriate option to fill in the blank. 

When She bid farewell to her parents, her eyes were full ……….. tears. 

(A) with 

(B) of 

(C) from 

(D) by 

Answer: (B) 

 

Q. 97 From the given options, identify the segment in the sentence which contains the grammatical error. He cannot speak Spanish because he was born and brought up in Spain. 

(A) born and brought up 

(B) He cannot speak 

(C) because he was 

(D) in Spain. 

Answer: (C) 

 

Q. 98 Select the word which means the same as the group of words given. 

A life history written by somebody else 

(A) Autobiography 

(B) Museology 

(C) Biography 

(D) Bibliography 

Answer: (C) 

 

Q. 99 Select the option that means the same as the given idiom. 

Spill the beans 

(A) To keep the secret 

(B) To be productive 

(C) To leak the secret 

(D) To be clumsy 

Answer: (C) 

 

Q. 100 Select the most appropriate word to fill in the blank. 

Let us go for a picnic today. The ………… is very pleasant. 

(A) atmosphere 

(B) climate 

(C) weather 

(D) season 

Answer: (C) 

SSC GD 14 Feb 2019 Shift-II Previous Year Paper

SSC GD 14th Feb 2019 Shift-II 

Reasoning 

Instructions 

For the following questions answer them individually 

Q. 1 In a certain code, WORKER is written as OKROKR. How will ASSUME be written in that code? 

(A) SUESUE 

(B) SUESVE 

(C) SUMSUM 

(D) SSESSE 

Answer: (A) 

 

Q. 2 Select the option that will correctly replace the question mark (?) in the given pattern. 

(A) 729 

(B) 125 

(C) 256 

(D) 225 

Answer: (D) 

 

Q. 3 Select the Venn diagram that best represents the given set of classes. Units, Watt, Volt 

Answer: (B) 

Explanation: 

Watt and Volt both are subset of Unit as both are measuring units. Therefore option (B) is the perfect representation of the given question. 

 

Q. 4 The statements below are followed by two conclusions labeled I and II. Assuming that the information in the statements is true, even if it appears at variance with generally established facts, decide which conclusion(s) logically and definitely follow(s) from the information given in the statements. 

Statements: 

1) Some mouses are whales. 

2) All whales are dogs. 

Conclusion: 

I. Some mouses are dogs. 

II. No mouse is dog. 

(A) Only conclusion I follows. 

(B) Both conclusions follow. 

(C) Neither conclusion I nor conclusion II follows. 

(D) Only conclusion II follows. 

Answer: (A) 

 

Q. 5 A square transparent sheet with a pattern is given. How will the pattern appear when the transparent sheet is folded along the dotted line? 

Answer: (D) 

 

Q. 6 Select the option that will correctly replace the question mark (?) in the series. 

9, 20, 42, 75, ? 

(A) 99 

(B) 129 

(C) 109 

(D) 119 

Answer: (D) 

 

Q. 7 Eight friends A, B, C, D, E, F, G and H are sitting around a circular table facing each other for a lunch. A is opposite F and third to the right of B. G is between F and D. H is to theleft of D. E is between C and A. Who is sitting opposite H? 

(A)

(B)

(C)

(D)

Answer: (A) 

 

Q. 8 The statements below are followed by two conclusions labeled I and II. Assuming that the information in the statements is true, even if it appears at variance with generally established facts, decide which conclusion(s) logically and definitely follow(s) from the information given in the statements. 

Statements: 

1) All knights are nuns. 

2) All nuns are men. 

Conclusion: 

I. All men are knights. 

II. All knights are men. 

(A) Either conclusion I or conclusion II follows. 

(B) Both conclusions follow. 

(C) Only conclusion II follows. 

(D) Only conclusion I follows. 

Answer: (C) 

 

Q. 9 Choose the option in which the figure marked ‘X’ is embedded. (Rotation is not allowed) 

Answer: (A) 

 

Q. 10 Choose the odd one out of the given options. 

(A) EKO 

(B) JIU 

(C) KVG 

(D) QMJ 

Answer: (A) 

 

Q. 11 Select the option that is related to the third term in the same way asthe second term is related to the first term. 

DEH : HIL :: GHL : ? 

(A) KJP 

(B) KPL 

(C) KLP 

(D) KLO 

Answer: (C) 

Explanation: 

DEH :HIL 

D+4 ->H 

E+4->I 

H+4->L 

Similarly 

GHL:? 

G+4->K 

H+4->L 

L+4->P 

Therefore, 

GHL:KLP 

 

Q. 12 Choose the option that most closely resembles the mirror image of the given word when mirror is placed at right side. 

Answer: (B) 

 

Q. 13 Choose the odd one out of the given options. 

(A) Colombia 

(B) Canada 

(C) Asia 

(D) China 

Answer: (C) 

 

Q. 14 Select a figure from amongst the four alternatives that when placed in the blank space (?) of figure X will complete the pattern. (Rotation is not allowed). 

Answer: (C) 

 

Q. 15 In a certain code, CIRCLE is written as 50. How will ALGAE be written in that code? 

(A) 57 

(B) 26 

(C) 23 

(D) 59 

Answer: (B) 

 

Q. 16 Seven friends O, P, Q, R, S, T and U are watching a movie sitting in a row. S is sitting at one extreme end. Q is sitting second to the right of S. P is sitting between O and Q.U is not sitting at any extreme end.R is sitting to the left of T. Who is sitting between P and R? 

(A)

(B)

(C)

(D)

Answer: (D) 

 

Q. 17 Select the option that will correctly replace the question mark (?) in the series. 

SNC, QPA, ORY, MTW, ? 

(A) KUU 

(B) KVV 

(C) KUV 

(D) KVU 

Answer: (D) 

 

Q. 18 Select the option that will correctly replace the question mark (?) in the series. 

20, 24, 40, 76, ? 

(A) 130 

(B) 120 

(C) 145 

(D) 140 

Answer: (D) 

 

Q. 19 Choose the option that would follow next in the given figure series. 

Answer: (C) 

 

Q. 20 J, K, L, M, N and O are six teachers. Each one teaches a different subject out of Hindi, English, Math, Science, Social Science and Arts, not necessarily in the same order. Each of them teaches on only one day, from Monday to Saturday, not necessarily in the same order. J teaches Arts on Saturday. L teaches neither English nor Social Science, but he teaches on Thursday. Wednesday is reserved for Maths taught by K. O teaches Science a day before N. Social Science is taught a day before Arts. Which subjectis taught on the day between Social Science and Math? 

(A) Science 

(B) Hindi 

(C) English 

(D) Arts 

Answer: (B) 

 

Q. 21 Select the option that is related to the third term in the same way as the second term is related to the first term. 

Aeroplane : Hanger :: Gun : ? 

(A) Shoot 

(B) Armoury 

(C) Human 

(D) Fire 

Answer: (B) 

 

Q. 22 Choose the odd number out of the given options. 

(A) 27 

(B) 625 

(C) 343 

(D) 125 

Answer: (B) 

 

Q. 23 Find out the two signs to be interchanged to make the following equation correct. 

30 + 28 × 3 − 16 ÷ 4 = −50 

(A) × and ÷ 

(B) + and − 

(C) − and × 

(D) − and ÷ 

Answer: (B) 

 

Q. 24 Select the option that is related to the third number in the same way as the second number is related to the first number. 

22 : 242 :: 18 : ? 

(A) 275 

(B) 169 

(C) 162 

(D) 221 

Answer: (C) 

 

Q. 25 Select the option that is related to the third term in the same wayas the second term is related to the first term. 

Chef : Knife :: Surgeon : ? 

(A) Scalpel 

(B) Grip 

(C) Chisel 

(D) Spade 

Answer: (A) 

General knowledge 

Instructions 

For the following questions answer them individually 

Q. 26 ‘Do or Die’ was the slogan of which famous movement in India? 

(A) Quit India Movement 

(B) Civil Disobedience Movement 

(C) Non-cooperation Movement 

(D) Rowlatt Movement 

Answer: (A) 

 

Q. 27 Which Indian city is the first to make it to the UNESCO World Heritage List? 

(A) Bhubaneswar 

(B) Indore 

(C) Bengaluru 

(D) Ahmedabad 

Answer: (D) 

 

Q. 28 Doddabetta is the highest peak of which of the following mountain ranges? 

(A) Vindhya 

(B) Satpura 

(C) Aravalli 

(D) Nilgiri 

Answer: (D) 

 

Q. 29 Which of the following statements about the National Human Rights Commission is correct? 

(A) It consists of 10 members. 

(B) Its powers are only recommendatory in nature. 

(C) Mumbai serves as its Headquarters. 

(D) Its chairman must be sitting judge in the Supreme Court. 

Answer: (B) 

 

Q. 30 The King Khalid International Airport serves which country? 

(A) Syria 

(B) Traq 

(C) Bahrain 

(D) Saudi Arabia 

Answer: (D) 

 

Q. 31 Which of the following is one of the criteria to represent the standard of living in a country? 

(A) Industrial growth 

(B) Poverty ratio 

(C) National income 

(D) Per capita income 

Answer: (D) 

 

Q. 32 Rapid integration between countries is known as: 

(A) globalisation 

(B) liberalisation 

(C) modernisation 

(D) privatisation 

Answer: (A) 

 

Q. 33 The term “double fault” is used in: 

(A) volleyball 

(B) tennis 

(C) snooker 

(D) kabbadi 

Answer: (B) 

 

Q. 34 ……… is widely used as a fuel and is a major component of compressed natural gas. 

(A) Propane 

(B) Octane 

(C) Methane 

(D) Butane 

Answer: (C) 

 

Q. 35 Which one of the following products received the first geographical indication (GI) in India? 

(A) Mizo Chilli 

(B) Ratlami Sev 

(C) Joha Rice 

(D) Darjeeling Tea 

Answer: (D) 

 

Q. 36 Where did Babur defeat Ibrahim Lodhi in 1526? 

(A) Haldighati 

(B) Kalinjar 

(C) Chausa 

(D) Panipat 

Answer: (D) 

 

Q. 37 Which temples are well-known for their Nagara-style of architecture having erotic sculptures? 

(A) Khajuraho 

(B) Somnath 

(C) Hampi 

(D) Meenakshi 

Answer: (A) 

 

Q. 38 The temperate grasslands of South Africa are called: 

(A) Downs 

(B) Praries 

(C) Pampas 

(D) Velds 

Answer: (D) 

 

Q. 39 The prestigious Laureus World Sports Awards was given to the Indian athlete: 

(A) Anirban Chatterji 

(B) Pankaj Advani 

(C) Dutee Chand 

(D) Vinesh Phogat 

Answer: (D) 

 

Q. 40 The term ‘mutation’ primarily refers to: 

(A) genetic migration 

(B) genetic line 

(C) genetic correction 

(D) genetic mistake 

Answer: (D) 

 

Q. 41 Which of the following countries won the Men’s Hockey World Cup 2018? 

(A) Belgium 

(B) New Zealand 

(C) Argentina 

(D) Pakistan 

Answer: (A) 

 

Q. 42 Panchayati Raj institutions came into existence under the: 

(A) 42nd and 43rd Amendment Acts 

(B) 86th and 87th Amendment Acts 

(C) 63rd and 64th Amendment Acts 

(D) 73rd and 74th Amendment Acts 

Answer: (D) 

 

Q. 43 Which IUCN document lists endangered species of flora and fauna? 

(A) Blue List 

(B) Green List 

(C) Red List 

(D) Black List 

Answer: (C) 

 

Q. 44 Who among the following introduced the practice of Sijda (prostration) and Paibos (kissing the feet of monarch) in the court? 

(A) Allauddin Khalji 

(B) Firoz Shah Tughluq 

(C) Muhammad bin Tughluq 

(D) Ghiyasuddin Balban 

Answer: (D) 

 

Q. 45 ……… is a microfinance organisation and community development bank founded in Bangladesh which makes small loans to the impoverished without requiring collateral. 

(A) Reserve bank 

(B) Federal bank 

(C) Cooperative bank 

(D) Grameen bank 

Answer: (D) 

 

Q. 46 Which of the following is NOT a sustainable development goal targeted to be achieved by 2030? 

(A) Gender Equality 

(B) Zero Hunger 

(C) Good Health and Wellbeing 

(D) Space Research 

Answer: (D) 

 

Q. 47 A………… measures the humidity of the air. 

(A) hygrometer 

(B) micrometer 

(C) thermometer 

(D) hydrometer 

Answer: (A) 

 

Q. 48 Which household waste has an excellent recycling potential? 

(A) Vegetable scraps 

(B) Metal 

(C) Plastic 

(D) Rubber 

Answer: (A) 

 

Q. 49 World Energy Out look is released by the: 

(A) International Energy Tribunal 

(B) International Energy Agency 

(C) World Economic Forum 

(D) International Energy Organisation 

Answer: (B) 

 

Q. 50 According to the 2011 census, the total percentage of Muslims in India is : 

(A) 10.2 

(B) 12.4 

(C) 16.6 

(D) 14.2 

Answer: (D) 

Quant 

Instructions 

For the following questions answer them individually 

Q. 51 A sum of ₹ 11700 becomes ₹ 16848 in 2 years at the rate of compound interest. If the interest is compounded annually, then what will be the rate of interest? 

(A) 20 % 

(B) 15 % 

(C) 17.5 % 

(D) 25 % 

Answer: (A) 

 

Q. 52 A sum of ₹ 4650 is lent at simple interest. If the rate of interest is 7.5% per annum, then what will be the simple interest for 4 years? 

(A) ₹ 1395 

(B) ₹ 1295 

(C) ₹ 1495 

(D) ₹ 1300 

Answer: (A) 

 

Q. 53 Pipe C can fill a tank in 12 hours and pipe D can fill the same tank in 40 hours. In how many hours both pipe C and D together can fill the same tank? 

(A) 60/7 hours 

(B) 60/11 hours 

(C) 120/13 hours 

(D) 120/11 hours 

Answer: (C) 

 

Q. 54 The length, breadth and height of a solid cuboid is 14 cm, 12 cm and 8 cm respectively. If cuboid is melted to form identical cubes of side 2 cm, then what will be the number of identical cubes? 

(A) 168 

(B) 144 

(C) 156 

(D) 128 

Answer: (A) 

 

Q. 55 The average weight of all the vehicles in a parking is 4000 kg. The average weight of 12 vehicles is 6000 kg. Average weight of the remaining vehicles is 3000 kg. What is the total number of vehicles in the parking? 

(A) 40 

(B) 36 

(C) 20 

(D) 30 

Answer: (B) 

Explanation: 

Let the total number of vehicles in the parking be x. 

Then, Total weight of all vehicles = 4000x kg 

Given, Average weight of 12 vehicles = 6000 kg 

Total weight of 12 vehicles = 72000 kg 

Remaining vehicles = x-12 

Given, Average weight of (x-12) vehicles = 3000 kg 

Total weight of (x-12) vehicles = 3000*(x-12) = 3000x-36000 kg 

Then, 72000 + 3000x – 36000 = 4000x 

=> 1000x = 36000 => x = 36 

Therefore, Number of vehicles in the parking = 36. 

 

Q. 56 What is the value of 80 ÷ 40 − 10 − 5 × 4 of (⅓  ÷ 10/3) ? 

(A) -12 

(B) 40/3

(C) 22/5

(D) -10 

Answer: (D) 

 

Q. 57 The total weight of 12 boys and 8 girls is 1080 kg. If the average weight of boys is 50 kg, then what will be average weight of girls? 

(A) 55 kg 

(B) 50 kg 

(C) 60 kg 

(D) 45 kg 

Answer: (C) 

 

Q. 58 The diagonal of a square is 14 cm. What will be the length of the diagonal of the square whose area is double of the area of first square? 

(A) 28√2 cm 

(B) 14√2 cm 

(C) 28 cm 

(D) 21√2 cm 

Answer: (B) 

 

Q. 59 The line chart given below shows the production (in 10000 s) of cars in 7 different months. 

Production of cars in month M4 is what percent of the production of cars in month M2? 

(A) 125% 

(B) 75% 

(C) 66.66% 

(D) 83.33% 

Answer: (B) 

 

Q. 60 What is the value of [88 − 44 ÷ (22 × 4) of (½ − ¼ ÷ ⅛ )]?? 

(A) 265/3

(B) 703/9

(C) 514/9

(D) 711/3

Answer: (A) 

 

Q. 61 The Least Common Multiple and Highest Common Factor of two numbers are 60 and 3 respectively. If their difference is 3, then what will be the sum of these two numbers? 

(A) 24 

(B) 35 

(C) 27 

(D) 21 

Answer: (C) 

Explanation: 

Let the two numbers be a and b. 

We know that the product of two numbers = Product of their LCM and HCF ab = 3*60 = 180 

Given, a – b = 3 

We know that 

(a − b)2 = (a + b)2 − 4ab 

32 = (a + b)2 − 4 × 180 

9 = (a + b)2 − 720 

=> (a + b)2 = 729 => a + b = 27 

Therefore, Sum of the two numbers = 27 

 

Q. 62 Selling price and cost price of an article are in the ratio of 7 : 5. What will be the profit/loss percentage? 

(A) 40% profit 

(B) 28.5% profit 

(C) 20% loss 

(D) 14.28% loss 

Answer: (A) 

 

Q. 63 The length of a rectangle is 16 cm. If the length of diagonal is 20 cm, then what will be the breadth of the rectangle? 

(A) 8 cm 

(B) 12 cm 

(C) 10 cm 

(D) 14 cm 

Answer: (B) 

 

Q. 64 The bar chart given below shows the annual production (in 1000 tonnes) of coffee of a country for years 2009 to 2013.

What is the average production (in 1000 tonnes) of coffee from year 2009 to 2011? 

(A) 1366.67 

(B) 1166.67 

(C) 1233.33 

(D) 1466.67 

Answer: (B) 

 

Q. 65 Manish goes to his hometown at the speed of 40 km/hr and returns to his home at the speed of Y km/hr. Distance between his home and hometown is 360 km. If he takes total 21 hours, then what is the value of Y? 

(A) 25 km/hr 

(B) 28 km/hr 

(C) 30 km/hr 

(D) 33 km/hr 

Answer: (C) 

 

Q. 66 P and Q together can complete a work in 20 days. If P alone can complete the same work in 36 days, then in how many days Q alone can complete the same work? 

(A) 48 days 

(B) 42 days 

(C) 45 days 

(D) 51 days 

Answer: (C) 

Explanation: 

Let the total work be 180 units (LCM of 20 and 36) 

Efficiency of P+Q = 180/20 = 9 units per day 

Efficiency of P = 180/36 = 5 units per day 

Then, Efficiency of Q = 9-5 = 4 units per day 

Therefore, Q can do the work together in 180/4 = 45 days 

 

Q. 67 The bar chart given below shows the annual production (in 1000 tonnes) of coffee of a country for years 2009 to 2013.

The production for the year 2011 is what percent of the production for the year 2009? 

(A) 157.5% 

(B) 175.25% 

(C) 187.5% 

(D) 205.25% 

Answer: (C) 

 

Q. 68 Four numbers are in the ratio of 2 : 5 : 3 : 8 respectively. If the sum of these four numbers is 432, then what is the sum of first and fourth number? 

(A) 192 

(B) 216 

(C) 240 

(D) 232 

Answer: (C) 

Explanation: 

Let the four numbers be 2x,5x,3x and 8x. 

Sum of four numbers = 2x+5x+3x+8x = 18x 

Given, 18x = 432 => x = 24 

Then, Sum of first and last numbers = 2x+8x = 10x = 10*24 = 240. 

 

Q. 69 The length of platform is double of the length of train. Speed of the train is 144 km/hr.If train crosses the platform in 30 seconds, then what is the length of the platform? 

(A) 600 metres 

(B) 800 metres 

(C) 500 metres 

(D) 400 metres 

Answer: (B) 

 

Q. 70 The ratio of number of children, women and men in colony is 6 : 4 : 3 respectively and the colony has at least 200 members. What will be minimum number of children in the colony? 

(A) 48 

(B) 72 

(C) 64 

(D) 96 

Answer: (D) 

Explanation: 

Given, Children : Women : Men = 6 : 4 : 3 

Let number of children be 6x 

Number of women be 4x 

Number of men be 3x 

Total members in the colony = 6x+4x+3x = 13x 

Given, 13x = 200 

Here, 200 is not divisible by 13. So, Total members can be taken as 208. 

13x = 208 

=> x = 16 

Therefore, Number of children = 6x = 6*16 = 96. 

 

Q. 71 The marked price of an article is ₹ 1200 and discount offered is 31%. If profit is 15%, then what will be the cost price? 

(A) ₹640 

(B) ₹704 

(C) ₹748 

(D) ₹720 

Answer: (D) 

 

Q. 72 If the length of a rectangle increases by 50% and the breadth decreases by 25%, then what will be the percent increase in its area? 

(A) 15% 

(B) 17.5% 

(C) 12.5% 

(D) 25% 

Answer: (C) 

Explanation: 

Given, Increase in Length = 50% Decrease in Breadth = 25% 

50 × 25 

= 50 − 25 − 100

= 25 − 12.5 = 12.5 

Then, Overall Change in Area % 

 

Q. 73 Selling price of first article is ₹ 470 and cost price of second article is ₹ 470. If there is a loss of 20% on first article and profit of 20% on second article, then what will be the overall profit or loss percentage? 

(A) 2.22% loss 

(B) 4% profit 

(C) No profit no loss 

(D) 1.80% loss 

Answer: (A) 

 

Q. 74 Teena, Reena and Sheena start a business with investment of respectively ₹ 24000, ₹ 28000 and ₹ 20000. Teena invests for 8 months, Reena invest for 10 months and Sheena invests for one year. If the total profit at the end of year is ₹ 25810, then what is the share of Teena? 

(A) ₹6960 

(B) ₹10150 

(C) ₹7940 

(D) ₹8700 

Answer: (A) 

 

Q. 75 What is the median of the given data? 

1, 3, 6, 2, 5, 8, 3, 8, 2 

(A)

(B)

(C)

(D) 2.5 

Answer: (A) 

Explanation: 

Arranging the given numbers in ascending order: 1,2,2,3,3,5,6,8,8 

Median is the middle most number in the series which is 3. 

English 

Instructions 

For the following questions answer them individually 

 

Q. 76 Select the most appropriate option to fill in the blank. 

Gita won’t pass the examination and …………. will Rita. 

(A) nor 

(B) or 

(C) either 

(D) neither 

Answer: (D) 

 

Q. 77 Select the most appropriate option to substitute the underlined segment in the given sentence.If there is no need to substitute it, select No Improvement. 

Tourists visiting Leh take home photos and memories but leave back a mountain of waste. 

(A) leave away 

(B) left back 

(C) No improvement 

(D) leave behind 

Answer: (D) 

Instructions 

In the following passage some words have been deleted. Fill in the blanks with the help of the alternatives given. 

Passage: 

The Second Anglo-Maratha Warhad shattered the (1)……… of the Maratha chiefs, but not theirspirit. The loss of their freedom caused them deep (2)……… They madea last (3)……….. attempt to regain their independencein 1817. The lead in organising a united front of the Maratha chiefs (4)…….. by the Peshwa who was smarting under the (5)……….. contro! exercised by the British Resident. 

 

Q. 78 Select the most appropriate option that will fill in the blank number 1. 

(A) patience 

(B) dignity 

(C) power 

(D) time 

Answer: (C) 

 

Q. 79 Select the most appropriate option that will fill in the blank number 2. 

(A) agony 

(B) distress 

(C) fear 

(D) depression 

Answer: (A) 

 

Q. 80 Select the most appropriate option that will fill in the blank number 3. 

(A) violent 

(B) massive 

(C) desperate 

(D) feeble 

Answer: (C) 

 

Q. 81 Select the most appropriate option that will fill in the blank number 4. 

(A) was taking 

(B) was taken 

(C) had taken 

(D) were taken 

Answer: (B) 

 

Q. 82 Select the most appropriate option that will fill in the blank number 5. 

(A) cruel 

(B) orthodox 

(C) pleasant 

(D) rigid 

Answer: (D) 

Instructions 

For the following questions answer them individually 

 

Q. 83 Select the synonym of the given word. 

AGILE 

(A) Aggressive 

(B) Active 

(C) Dull 

(D) Lazy 

Answer: (B) 

 

Q. 84 Select the correctly spelt word. 

(A) Gratefull 

(B) Greatfull 

(C) Greatful 

(D) Grateful 

Answer: (D) 

 

Q. 85 Select the antonym of the given word. 

HIDEOUS 

(A) Beautiful 

(B) Secretive 

(C) Detestable 

(D) Ugly 

Answer: (A) 

 

Q. 86 Select the antonym of the given word. 

BLESS 

(A) Purify 

(B) Sanctify 

(C) Wish 

(D) Curse 

Answer: (D) 

 

Q. 87 From the given options, identify the segmentin the sentence which contains the grammatical error. Ram alwaystries to do things very carefully and made sure he does them correctly. 

(A) to do things 

(B) Ram always tries 

(C) does them 

(D) and made sure 

Answer: (D) 

 

Q. 88 From the given options, identify the segment in the sentence which contains the grammatical error. If they had a playground here, they can play after their school is over. 

(A) after their 

(B) If they had 

(C) school is over 

(D) they can play 

Answer: (D) 

 

Q. 89 Select the correctly spelt word. 

(A) Occation 

(B) Occassion 

(C) Occasion 

(D) Ocassion 

Answer: (C) 

 

Q. 90 Select the option that means the same as the given idiom. 

A red letter day 

(A) (A) sad day 

(B) (A) boring day 

(C) (A) memorable day 

(D) (A) painful day 

Answer: (C) 

 

Q. 91 Select the word which means the same as the group of words given. 

Someone who leaves one’s country to settle in another country. 

(A) Tourist 

(B) Emigrant 

(C) Foreigner 

(D) Explorer 

Answer: (B) 

 

Q. 92 Select the most appropriate word to fill in the blank. 

Just are the ….. of God. 

(A) ways 

(B) deeds 

(C) plans 

(D) tricks 

Answer: (A) 

 

Q. 93 Select the synonym of the given word. 

MELODIOUS 

(A) Rhythmic 

(B) Rattling 

(C) Tuneful 

(D) Doleful 

Answer: (C) 

 

Q. 94 Select the option that means the same as the given idiom. 

To eat humble pie 

(A) To deny desperately 

(B) To defend oneself 

(C) To accept error 

(D) To be aggressive 

Answer: (C) 

 

Q. 95 From the given options, identify the segmentin the sentence which contains the grammatical error. I said you to prepare the article yesterday but you didn’t. 

(A) the article 

(B) I said you 

(C) you didn’t 

(D) to prepare 

Answer: (B) 

 

Q. 96 Select the most appropriate option to fill in the blank. 

You can visit me on ………. at 7 o’clock. 

(A) tomorrow 

(B) next month 

(C) today 

(D) Sunday 

Answer: (D) 

 

Q. 97 Select the most appropriate option to substitute the underlined segment in the given sentence. If there is no need to substitute it, select No Improvement. 

Yesterday eleven coaches of a train headed for Delhi was derailed at Vaishali district of Bihar. 

(A) was derailed in 

(B) were derailed in 

(C) No improvement 

(D) were derail at 

Answer: (B) 

 

Q. 98 Select the most appropriate word to fill in the blank. 

…………. are to be avoided: they lead to conflict. 

(A) Questions 

(B) Statements 

(C) Arguments 

(D) Decisions 

Answer: (C) 

 

Q. 99 Select the most appropriate option to substitute the underlined segment in the given sentence. If there is no need to substitute it, select No Improvement. 

On inspection a hundred year old bridge was find being too weak to take the load of lakhs of vehicles everyday. 

(A) was being found 

(B) No improvement 

(C) was found to be 

(D) has to be found 

Answer: (C) 

 

Q. 100 Select the word which means the same as the group of words given. 

One who is present everywhere 

(A) Omnipotent 

(B) Omniscient 

(C) Omnipresent 

(D) Omnivorous 

Answer: (C) 

SSC GD 14 Feb 2019 Shift-I Previous Year Paper

SSC GD 14th Feb 2019 Shift-I 

Reasoning 

Instructions 

For the following questions answer them individually 

Q. 1 J, K, L, M, N and O are six teachers. Each one teaches a different subject out of Hindi, English, Math, Science, Social Science and Arts, not necessarily in the same order. Each of them teaches on only one day, from Monday to Saturday, not necessarily in the same order. J teaches Arts on Saturday. L teaches neither English nor Social Science, but he teaches on Thursday. Wednesday is reserved for Maths taught by K. O teaches Science a day before N. Social Science is taught a day before Arts. Which subject is taught on the day between Science and Math? 

(A) Hindi 

(B) Arts 

(C) Science 

(D) English 

Answer: (D) 

 

Q. 2 Choose the option that most closely resembles the mirror image of the given word when the mirror is placed at the right side.

Answer: (B) 

 

Q. 3 Select a figure from amongst the four alternatives that when placed in the blank space (?) of figure X will complete the pattern. (Rotation is not allowed). 

Answer: (A) 

 

Q. 4 Select the option that is related to the third term in the same wayas the second term is related to the first term. 

Chapter : Book :: Arc : ? 

(A) Cube 

(B) Triangle 

(C) Square 

(D) Circle 

Answer: (D) 

 

Q. 5 A square transparent sheet with a pattern is given. How will the pattern appear when the transparent sheet is folded along the dotted line? 

Answer: (B) 

 

Q. 6 Select the option that will correctly replace the Q. mark (?) in the series. 

16, 20, 36, 72, ? 

(A) 126 

(B) 116 

(C) 136 

(D) 146 

Answer: (C) 

 

Q. 7 Eight friends A, B, C, D, E, F, G and H are sitting arounda circular table facing each other for a lunch. A is opposite F and third to the right of B. G is between F and D.H isto theleft of D. E is between C and A. Whois sitting fourth to the left of A? 

(A)

(B)

(C)

(D) C

Answer: (C) 

 

Q. 8 Select the option that is related to the third term in the same wayas the second term is related to the first term. 

Proud : Haughty : Fallacy : ? 

(A) Ego 

(B) War 

(C) Fall 

(D) Illusion 

Answer: (D) 

 

Q. 9 Select the option that is related to the third number in the same way as the second number is related to the first number. 

32 : 256 :: 16 : ? 

(A) 128 

(B) 122 

(C) 144 

(D) 66 

Answer: (A) 

 

Q. 10 Choose the odd one out of the given options. 

(A) Ottawa 

(B) Thimphu 

(C) Beijing 

(D) Chile 

Answer: (D) 

 

Q. 11 In a certain code, BICYCLE is written as 59. How will WINDOW be written in that code? 

(A) 88 

(B) 91 

(C) 89 

(D) 77 

Answer: (A) 

 

Q. 12 Choose the option that would follow next in the given figure series. 

Answer: (A) 

 

Q. 13 Select the option that will correctly replace the Q. mark (?) in the series. 

8, 19, 41, 74, ? 

(A) 108 

(B) 98 

(C) 118 

(D) 128 

Answer: (C) 

 

Q. 14 Choose the odd one out of the given options. 

(A) ILP 

(B) JHU 

(C) FJU 

(D) THI 

Answer: (B) 

 

Q. 15 Choose the option in which the figure marked ‘X’ is embedded. (Rotation is not allowed) 

Answer: (C) 

 

Q. 16 Choose the odd number out of the given options. 

(A) 651 

(B) 571 

(C) 561 

(D) 471 

Answer: (B) 

 

Q. 17 In a certain code, BOTANY is written as OBATYN. How will OOLOGY be written in that code? 

(A) OOOLGY 

(B) OLOOYG 

(C) OOLOYG 

(D) OOOLYG 

Answer: (D) 

 

Q. 18 The statements below are followed by two conclusions labeled I and II. Assuming that the information in the statements is true, even if it appears at variance with generally established facts, decide which conclusion(s) logically and definitely follow(s) from the information given in the statements. 

Statements: 

1) Some forces are volumes. 

2) Some currents are volumes. 

Conclusion: 

I. Some volumes are forces. 

II. Some volumes are currents. 

(A) Only conclusion II follows. 

(B) Neither conclusion I nor conclusion II follows. 

(C) Both conclusions follow. 

(D) Only conclusion I follows. 

Answer: (C) 

 

Q. 19 The statements below are followed by two conclusions labeled I and II. Assuming that the information in the statements is true, even if it appears at variance with generally established facts, decide which conclusion(s) logically and definitely follow(s) from the information given in the statements. 

Statements: 

1) Some wines are chairs. 

2) All chairs are drums. 

Conclusion 

I. All chairs are wines. 

II. Some drums are wines. 

(A) Either conclusion I or conclusion II follows. 

(B) Only conclusion I follows. 

(C) Only conclusion II follows. 

(D) Both conclusions follow. 

Answer: (C) 

 

Q. 20 Select the Venn diagram that best represents the given set of classes. 

Currencies, Yuan, Baht 

Answer: (C) 

 

Q. 21 Seven friends O, P, Q, R, S, T and U are watching a moviesitting in a row.S is sitting at one extreme end. Q is sitting second to the right ofS.P is sitting between O and Q.U is notsitting at any extreme end. is sitting to the left of T. Who is sitting between S and Q? 

(A)

(B)

(C)

(D)

Answer: (A) 

 

Q. 22 Select the option that will correctly replace the Q. mark (?) in the series. 

RMB, POZ, NQX, LSV, ? 

(A) JUT 

(B) JVT 

(C) JUS 

(D) JVS 

Answer: (A) 

 

Q. 23 Find out the two signs to be interchanged to make the following equation correct. 

21 + 5 × 2 − 21 ÷ 3 = 12 

(A) + and − 

(B) − and ÷ 

(C) − and × 

(D) × and ÷ 

Answer: (C) 

 

Q. 24 Select the option that will correctly replace the Q. mark (?) in the given pattern. 

(A)

(B)

(C) 11 

(D) 13 

Answer: (A) 

 

Q. 25 Select the option that is related to the third term in the same way as the second term is related to the first term. 

GKM : FIO: : EIK : ? 

(A) DGN 

(B) DFM 

(C) DGM 

(D) DHM 

Answer: (C) 

General knowledge 

Instructions 

For the following questions answer them individually 

Q. 26 Ashoka appointed religious officials, known as: 

(A) Dhamma-mahamatta 

(B) Dhammadhiraja 

(C) Dhammadhikari 

(D) Dhammadhyaksha 

Answer: (A) 

 

Q. 27 Bank rate is decided by which of the following agencies? 

(A) Securities and Exchange Board of India 

(B) Reserve Bank of India 

(C) State Bank of India 

(D) Ministry of Finance 

Answer: (B) 

 

Q. 28 The cabinet has raised the Minimum Support Price of wheat for 2018-19 by: 

(A) ₹ 105 per quintal 

(B) ₹ 202 per quintal 

(C) ₹ 309 per quintal 

(D) ₹ 600 per quintal 

Answer: (A) 

 

Q. 29 In which year was the Earth summit of Rio de Janeiro held? 

(A) 1991 

(B) 1993 

(C) 1992 

(D) 1990 

Answer: (C) 

 

Q. 30 Pandit Bhimsen Joshi is associated with which tradition of classical music? 

(A) Qawwali 

(B) Thumri 

(C) Carnatic 

(D) Hindustani 

Answer: (D) 

 

Q. 31 Muscle cramp is caused by: 

(A) water loss during sweating 

(B) fatiguing muscles respiring aerobically 

(C) increased blood circulation 

(D) accumulation of lactic acid 

Answer: (D) 

 

Q. 32 Which of the following languages is NOT a part of the Eighth Schedule of the Constitution? 

(A) Dogri 

(B) Bodo 

(C) Persian 

(D) Santhal 

Answer: (C) 

 

Q. 33 Which of the following is NOT in the jurisdiction of the Supreme Court of India? 

(A) Appellate 

(B) Original 

(C) Advisory 

(D) Financial 

Answer: (D) 

 

Q. 34 The percolation rate of water is the least in ……….. soil. 

(A) clayey 

(B) sandy 

(C) gravel 

(D) loamy 

Answer: (A) 

 

Q. 35 The Jnanpith Award is given for excellence in which of the following fields? 

(A) Medicine 

(B) Social work 

(C) Literature 

(D) Cinema 

Answer: (C) 

 

Q. 36 The 2018 Commonwealth Games were held in: 

(A) Gold Coast 

(B) London 

(C) Doha 

(D) Brazil 

Answer: (A) 

 

Q. 37 Which one of the following countries is NOT a member of the BASIC grouping? 

(A) South Africa 

(B) China 

(C) Brazil 

(D) Kenya 

Answer: (D) 

 

Q. 38 The 86th Constitution Amendment Act, 2002 inserted which of the following articles in the Constitution of India? 

(A) 21-A

(B) 39-A 

(C) 20-A 

(D) 56-C 

Answer: (A) 

 

Q. 39 Which of the following heritage sites shows the Dravidian style of architecture? 

(A) Konark 

(B) Bodh Gaya 

(C) Akshardham 

(D) Hampi 

Answer: (D) 

 

Q. 40 ……….. was the founder of the Shuddhi Movement. 

(A) Swami Dayanand Saraswati 

(B) Swami Vivekananda 

(C) Ishwar Chandra Vidyasagar 

(D) Raja Ram Mohan Roy 

Answer: (A) 

 

Q. 41 Which among the following CANNOT be used as indicators? 

(A) China Rose 

(B) Phenolphthalein 

(C) Common salt 

(D) Turmeric 

Answer: (C) 

 

Q. 42 The Nagarjuna Sagar Dam is built across the river: 

(A) Pamba 

(B) Mahanadi 

(C) Krishna 

(D) Cauvery 

Answer: (C) 

 

Q. 43 The Jataka tales are associated with which of the following sects? 

(A) Lingayat 

(B) Shaivism 

(C) Jainism 

(D) Buddhism 

Answer: (D) 

 

Q. 44 Mirabai Chanu won a Gold medal at which of the following events held in Anaheim, United States? 

(A) World Snooker Championships 

(B) World Archery Championships 

(C) World Weightlifting Championships 

(D) World Tennis Championships 

Answer: (C) 

 

Q. 45 ……… refers to a deposit into a bank accountor a financial institution with no specified maturity date. 

(A) demand deposit 

(B) fixed deposit 

(C) recurring deposit 

(D) current deposit 

Answer: (D) 

 

Q. 46 The ‘Gol Gumbaz’ is located in: 

(A) Bijapur 

(B) Bidar 

(C) Daulatabad 

(D) Hyderabad 

Answer: (A) 

 

Q. 47 According to Census 2011, ………. has the highest literacy rate. 

(A) Mizoram 

(B) Goa 

(C) Kerala 

(D) Karnataka 

Answer: (C) 

 

Q. 48 ‘Adopt a Heritage: Apni Dharohar, Apni Pehchaan’, is the initiative of ……… 

(A) Indian Council of Historical Research 

(B) Ministry of Tourism 

(C) Archaeological Survey of India 

(D) Ministry of Culture 

Answer: (B) 

 

Q. 49 Land and sea breeze are caused due to …………. 

(A) convection 

(B) acclimatisation 

(C) radiation 

(D) conduction 

Answer: (A) 

 

Q. 50 What is the role of mucus secreted by the stomach? 

(A) To protect the lining of the stomach 

(B) To digest fats 

(C) To kill the germs in the food 

(D) To digest the proteins 

Answer: (A) 

Quant 

Instructions 

For the following questions answer them individually 

Q. 51 P, Q and R can complete a work in 30 days, 15 days and 20 days respectively. P works on first day, then Q works on second day and then R works on third day and so on. In how many days the work will be completed? 

(A) 20 days 

(B) 21 days 

(C) 22 days 

(D) 19 days 

Answer: (A) 

Explanation: 

Let the total work be 60 units (LCM of 30,15 and 20) 

Efficiency of P = 60/30 = 2 units per day 

Efficiency of Q = 60/15 = 4 units per day 

Efficiency of R = 60/20 = 3 units per day 

Then, 9 units of work will be completed in 3 days. 

=> 54 units of work will be completed in 18 days. 

Next 2 units of work will be completed by P in 1 day. 

Remaining 4 units of work will be completed by Q in 1 day. 

Therefore, Time taken to complete 60 units of work = 18+1+1 = 20 days. 

 

Q. 52 If length, breadth and height of cuboids are increases by 10%, 20% and 15% respectively, then what will be the percentage change in its volume? 

(A) 47.4% 

(B) 45% 

(C) 51.8% 

(D) 54.2% 

Answer: (C) 

 

Q. 53 What is the value of (24 ÷ 4 − ⅔ of 21/8 ÷ ¼)?

(A) −1 

(B) -96/15

(C) 91/16

(D) -8

Answer: (A) 

 

Q. 54 If the average of five consecutive odd numbers is 17, then what will be the highest number among them ? 

(A) 21 

(B) 25 

(C) 19 

(D) 23.5 

Answer: (A) 

 

Q. 55 The length of a rectangle is 24 cm. If the length of diagonal is 26 cm, then what will be the breadth of the rectangle? 

(A) 12 cm 

(B) 14 cm 

(C) 15 cm 

(D) 10 cm 

Answer: (D) 

 

Q. 56 If selling price of an article is ₹ 432 and profit is 35%, then what will be the cost price of the article? 

(A) ₹480.40 

(B) ₹320 

(C) ₹583.20 

(D) ₹368 

Answer: (B) 

 

Q. 57 A train covers a certain distance at a speed of 45 m/s in 15 minutes. How much time it will take to cover the same distance at the speed of 60 m/s? 

(A) 3.25 minutes 

(B) 4.75 minutes 

(C) 6.75 minutes 

(D) 11.25 minutes 

Answer: (D) 

 

Q. 58 What is average of 410, 475, 525, 560 and 720? 

(A) 561 

(B) 542 

(C) 538 

(D) 526 

Answer: (C) 

 

Q. 59 The diagonal of a square is 10 cm. What will be the length of the diagonal of the square whose area is double of the area of first square? 

(A) 10√2 cm 

(B) 8√2 cm 

(C) 20√2 cm 

(D) 20 cm 

Answer: (A) 

 

Q. 60 What will be the difference between the compound interest (interest is compounded annually) and simple interest on a sum of ₹ 6400 at the rate of 10% per annum for 2 years? 

(A) ₹ 90 

(B) ₹ 72 

(C) ₹ 128 

(D) ₹ 64 

Answer: (D) 

 

Q. 61 The radii of two circles are 20 cm and 15 cm respectively. If a third circle has an area which is equal to the sum of the areas of the two given circles, what will be the radius of the third circle? 

(A) 35 cm 

(B) 22 cm 

(C) 27 cm 

(D) 25 cm 

Answer: (D) 

 

Q. 62 A sum of ₹ 3460 is lent at simple interest. If the rate of interest is 5% per annum,then what will be the simple interest for 5 years? 

(A) ₹ 875 

(B) ₹ 865 

(C) ₹ 855 

(D) ₹ 845 

Answer: (B) 

Explanation: 

Given, Principal = Rs.3460 

Rate of interest = 5% per annum 

Time period = 5 years 

Then, Simple Interest for 1 year = 5% of 3460 = Rs.173 

Simple Interest for 5 years = 5*173 = Rs.865 

 

Q. 63 What is the value of 5/4 of 12/5 ÷ 4/25 + ½ × ¾ ?

(A) 149/4

(B) 153/8

(C) 149/8

(D) 153/16

Answer: (B) 

 

Q. 64 What will be the ratio of Highest common Factor and Least Common Multiple of two numbers 14 and 18? 

(A) 1 : 49 

(B) 1 : 27 

(C) 1 : 21 

(D) 1 : 63 

Answer: (D) 

Explanation: 

Given numbers are 14 and 18. 

LCM of 14 and 18 = 126 

HCF of 14 and 18 = 2 

Therefore, HCF : LCM = 2 : 126 = 1 : 63 

 

Q. 65 The bar chart given below shows the annual production (in 1000 tonnes) of coffee of a country for years 2009 to 2013.

What is the percentage increase in the production of coffee in the year 2011 as compared to year 2010? 

(A) 20% 

(B) 27.5% 

(C) 25% 

(D) 32.5% 

Answer: (C) 

 

Q. 66 The pie chart given below shows sale of different types of cars in a city for a given year. The total sale of cars in the city is 600000.

What is the sum of total number of cars sold of type T5, T6 and T3? 

(A) 232000 

(B) 228000 

(C) 245000 

(D) 252000 

Answer: (B) 

Explanation: 

Cars sold of type T3 = 13% of 6 lakhs 

Cars sold of type T5 = 17% of 6 lakhs 

Cars sold of type T6 = 8% of 6 lakhs 

Therefore, Total cars sold of type T3, T5 and T6 = 13% of 6 lakhs + 17% of 6 lakhs + 8% of 6 lakhs = 38% of 6 lakhs = 228000 

 

Q. 67 If two successive discounts of 25% and 20% respectively are given, then what will be the net discount percentage? 

(A) 42.5% 

(B) 45% 

(C) 50% 

(D) 40% 

Answer: (D) 

 

Q. 68 What is the mean of the given data? 

23, 24, 25, 26, 27, 28, 29, 30, 31, 32 

(A) 26.75 

(B) 27.69 

(C) 27.5 

(D) 26.12 

Answer: (C) 

 

Q. 69 A alone can complete a work in 10 days and B alone can complete the same work in 20 days. In how many days both 4 and B together can complete half of the total work? 

(A) 40/3 days

(B) 20/3 days 

(C) 10/3 days

(D) 25/3 days

Answer: (C) 

 

Q. 70 P and Q starts a business with investment of ₹ 28000 and ₹ 42000 respectively. P invests for 8 months and Q invests for one year.If the total profit at the end of the year is ₹ 21125, then what is the share of P? 

(A) ₹ 12625 

(B) ₹ 14625 

(C) ₹ 6500 

(D) ₹ 8750 

Answer: (C) 

 

Q. 71 The pie chart given below shows sale of different types of cars in a city for a given year. The total sale of cars in the city is 600000.

What is the difference in the number of cars sold of type T5 and T1? 

(A) 21000 

(B) 24000 

(C) 18000 

(D) 27000 

Answer: (C) 

Explanation: 

Number of cars sold of T1 = 20% of 6 lakhs 

Number of cars sold of T5= 17% of 6 lakhs 

Required difference = 20% of 6 lakhs – 17% of 6 lakhs = 3% of 6 lakhs = 18000 

 

Q. 72 Four numbers are in the ratio of 3 : 1 : 7 : 5 respectively. If the sum of these four numbers is 336, then what is the sum of first and fourth number? 

(A) 152 

(B) 172 

(C) 168 

(D) 146 

Answer: (C) 

Explanation: 

Let the four numbers be 3x,x,7x,5x 

Sum of the four numbers = 3x+x+7x+5x = 16x 

Given, 16x = 336 => x = 21 

Then, the numbers are, 

3x = 3*21 = 63 

x = 21 

7x = 7*21 = 147 

5x = 5*21 = 105 

Therefore, Sum of first and fourth numbers is 63+105 = 168. 

 

Q. 73 A train is moving at the speed of 20 m/sec. If the length of train is 540 metres, then how much time will it take to cross a pole? 

(A) 108 seconds 

(B) 81 seconds 

(C) 27 seconds 

(D) 54 seconds 

Answer: (C) 

 

Q. 74 The ratio of number of children, women and men in colony is 6 : 4 : 3 respectively and the colony has at least 200 members. What could be the minimum number of women in the colony? 

(A) 75 

(B) 80 

(C) 64 

(D) 20 

Answer: (C) 

Explanation: 

Let the number of children, women and men be 6x, 4x and 3x respectively. 

Then, Total members in the colony = 6x+4x+3x = 13x 

Given that the colony has at least 200 members. 

Since, 200 is not divisible by 13, Total members in the colony will be 208. 

13x = 208 => x = 16 

Therefore, Number of women = 4x = 4*16 = 64 

 

Q. 75 A person purchases bags at the rate of ₹ 128 per bag. If he sells bags on 20% profit, then what will be the selling price of 15 bags? 

(A) ₹ 1600 

(B) ₹ 2304 

(C) ₹ 2194 

(D) ₹ 1886 

Answer: (B) 

English 

Instructions 

For the following questions answer them individually 

Q. 76 Select the most appropriate option to substitute the underlined segment in the given sentence. If there is no need to substitute it, select No Improvement. 

My favourite novelist in all was Charles Dickens, who really left his mark on me. 

(A) No improvement 

(B) My favourite novelist of all 

(C) My favourite novelist between all 

(D) My favourite novelist at all 

Answer: (B) 

 

Q. 77 From the given options, identify the segment in the sentence which contains the grammatical error. The man greeted his secretary and told her that he need her help to complete the work immediately. 

(A) to complete the work 

(B) told her that 

(C) he need her help 

(D) greeted his secretary 

Answer: (C) 

 

Q. 78 Select the most appropriate option to substitute the underlined segment in the given sentence. If there is no need to substitute it, select No Improvement. 

Meeting my teacher after ten years was an unforgotten moment that I will always treasure. 

(A) a forgotten moment 

(B) an unforgettable moment 

(C) the unforgotten moment 

(D) No improvement 

Answer: (B) 

 

Q. 79 Select the most appropriate word to fill in the blank. 

Smoking is ……… to health. 

(A) tedious 

(B) serious 

(C) anxious 

(D) injurious 

Answer: (D) 

 

Q. 80 Select the antonym of the given word. 

STINGY 

(A) Generous 

(B) Skinny 

(C) Cheap 

(D) Mean 

Answer: (A) 

 

Q. 81 Select the most appropriate word to fill in the blank. 

Let us ……….. our heads together to solve the problem. 

(A) join 

(B) put 

(C) bring 

(D) combine 

Answer: (B) 

 

Q. 82 Select the option that means the same as the given idiom. 

To see eye to eye 

(A) To agree with another 

(B) To love another 

(C) To threaten another 

(D) To fight with another 

Answer: (A) 

 

Q. 83 Select the correctly spelt word. 

(A) Acheive 

(B) Achieve 

(C) Acheeve 

(D) Achiev 

Answer: (B) 

 

Q. 84 Select the most appropriate option to fill in the blank. 

In the United States of America two dozen people ……… due to the present cold wave. 

(A) have been killed 

(B) are being killed 

(C) has killed 

(D) has been killed 

Answer: (A) 

 

Q. 85 Select the most appropriate option to substitute the underlined segment in the given sentence. If there is no need to substitute it, select No Improvement. 

When you go by rail, make sure taking the express train that stops only at big stations. 

(A) make sure to taking 

(B) No improvement 

(C) make sure to take 

(D) making sure to take 

Answer: (C) 

 

Q. 86 Select the most appropriate option to fill in the blank. 

I think that childhood is a time when there are ………… ways to make life enjoyable. 

(A) many 

(B) less 

(C) little 

(D) more 

Answer: (A) 

 

Q. 87 From the given options, identify the segment in the sentence which contains the grammatical error. After finishing his tiring game, Rohan leaned by the wall. 

(A) his game 

(B) Rohan leaned 

(C) After finishing 

(D) by the wall 

Answer: (D) 

 

Q. 88 Select the synonym of the given word. 

IMPECCABLE 

(A) Imperfect 

(B) Flawless 

(C) Smooth 

(D) Clean 

Answer: (B) 

 

Q. 89 Select the option that means the same as the given idiom. 

To have the gift of the gab 

(A) To be eloquent 

(B) To be intelligent 

(C) To be boastful 

(D) To be gifted 

Answer: (A) 

 

Q. 90 Select the antonym of the given word. 

OPAQUE 

(A) Turbid 

(B) Transparent 

(C) Fogged 

(D) Murky 

Answer: (B) 

Instructions 

In the following passage some words have been deleted. Fill in the blanks with the help of the alternatives given. 

Passage: 

Without water(1)………. animal can survive.In desert regions the greatest (2)…….. to life is dying of thirst. But many creatures are able to make useof(3)………… little water that exists in arid areas. One of nature’s masterpieces (4)………… creatures equipped to (5)………… with desert life is the hardy camel. 

Q. 91 Select the most appropriate option that will fill in the blank number 1. 

(A) any 

(B) some 

(C) every 

(D) no 

Answer: (D) 

 

Q. 92 Select the most appropriate option that will fill in the blank number 2. 

(A) threat 

(B) warning 

(C) worry 

(D) requirement 

Answer: (A) 

 

Q. 93 Select the most appropriate option that will fill in the blank number 3. 

(A) the 

(B) that 

(C) very 

(D) what 

Answer: (A) 

 

Q. 94 Select the most appropriate option that will fill in the blank number 4. 

(A) besides 

(B) about 

(C) among 

(D) between 

Answer: (C) 

 

Q. 95 Select the most appropriate option that will fill in the blank number 5. 

(A) cope 

(B) resist 

(C) adapt 

(D) live 

Answer: (A) 

Instructions 

For the following questions answer them individually 

 

Q. 96 Select the word which means the same as the group of words given. 

A place where fights take place 

(A) Arsenal 

(B) Arena 

(C) Asylum 

(D) Aquarium 

Answer: (B) 

 

Q. 97 Select the word which means the same as the group of words given. 

A poem that expresses lament for the dead 

(A) Lyric 

(B) Ode 

(C) Elegy 

(D) Sonnet 

Answer: (C) 

 

Q. 98 From the given options, identify the segment in the sentence which contains the grammatical error. If it weren’t for the old man, we haven’t had anything to talk about. 

(A) we haven’t had 

(B) anything to talk 

(C) for the old man 

(D) If it weren’t 

Answer: (A) 

 

Q. 99 Select the synonym of the given word. 

SLENDER 

(A) Narrow 

(B) Wide 

(C) Slim 

(D) Weak 

Answer: (C) 

 

Q. 100 Select the correctly spelt word. 

(A) Permanant 

(B) Permanent 

(C) Pirmenent 

(D) Perminent 

Answer: (B) 

×

Hello!

Click one of our representatives below to chat on WhatsApp or send us an email to info@vidhyarthidarpan.com

×